IAS Baba Prelims 2018 Test 14 With Solutions

¡Supera tus tareas y exámenes ahora con Quizwiz!

QUESTION 66. Who of the following was called the Ambassador of Hindu - Muslim Unity? a) Mohammad Ali Jinnah b) Mahatma Gandhi c) Maulana Abul Kalam Azad d) Sarojini Naidu

Correct Answer: A Explanation Solution (a) Sarojini Naidu called Mohammad Ali Jinnah an Ambassador of Hindu - Muslim Unity. Think · Why did Jinnah Left Congress and went to London? · When did Jinnah roped in the Communal politics?

QUESTION 33. Which of the following statements are true about Round Table Conferences (RTC)? 1. Dr. B.R. Ambedkar attended all the three conferences. 2. The conferences were based on the recommendations of Muhammad Ali Jinnah and Simon commission report. 1. Dr B R Ambedkar demanded separate electorates for the 'untouchables' in the first Round Table Conference. 4. Mahatma Gandhi and Jawaharlal Nehru represented Indian National Congress at the second RTC. Select the correct code a) 1,2 and 3 b) 1,3 and 4 c) 2,3 and 4 d) 1 and 4

Correct Answer: A Explanation Solution (a) The Round Table Conferences were a series of three conferences conducted by the Labour Party-led British government to deliberate upon and bring about constitutional reforms in British India during 1930 - 32. There were three such conferences. The conferences were based on the recommendation of Muhammad Ali Jinnah to Lord Irwin, the then Viceroy of India and James Ramsay MacDonald, the then British Prime Minister, and the Simon Commission report . It was for the first time that the Indians and the British were meeting as 'equals'. First RTC · The First Round Table Conference was held between November 1930 and January 1931 at London. · Dr. B R Ambedkar demanded separate electorates for the 'untouchables' . • Tej Bahadur Sapru moved the idea of an All-India Federation. The Muslim League supported this. The princely states also supported this on the condition that their internal sovereignty is maintained. • Although many principles on reforms were agreed upon, not much was implemented and the Congress Party carried on its civil disobedience. The Indian National Congress decided not to participate in the conference. Many of the INC leaders were imprisoned due to their involvement in the civil disobedience movement. The Conference was regarded as a failure. • The British government understood the importance and the need for the Congress Party to make any decision on India's political future. Second RTC • The Second Round Table Conference was held between September and December 1931 at London. • The major difference between the first and the second conference was that the INC was participating in the second one. This was one of the results of the Gandhi-Irwin Pact. · Gandhi represented Indian National Congress and Sarojini Naidu represented Indian women. • The British decided to grant a communal award for representing minorities in India by providing for separate electorates for minority communities. Gandhi was against this. • In this conference, Gandhi and Ambedkar differed on the issue of separate electorates for the untouchables. Gandhi was against treating untouchables as separate from the Hindu community. This issue was resolved through the Poona Pact 1932. • The second round table conference was deemed a failure because of the many disagreements among the participants. While the INC claimed to speak for the whole of the country, other participants and leaders of other parties contested this claim. Third RTC • The Third Round Table Conference was held between November and December 1932 at London. • Only 46 delegates in total took part in this conference. The INC and the Labour Party decided not to attend it. (The INC wasn't invited). • Not much was achieved in this conference also. • The recommendations of this conference were published in a White Paper in 1933 and later discussed in the British Parliament. The recommendations were analysed and the Government of India Act of 1935 was passed on its basis. Dr. B.R. Ambedkar attended all three conferences. Do you know? · Poona Pact refers to an agreement between B. R. Ambedkar and M. K. Gandhi on the reservation of electoral seats for the depressed classes in the legislature of then British India. THINK! · Communal Award

QUESTION 13. Which of the following statements is/are true regarding Civil disobedience movement? 1. Before launching the movement, Gandhi presented 'Eleven Demands' to the government. 2. C. Rajagopalachari led the march to defy salt laws in Malabar. 3. No- chowkidara tax campaigns were started in Ryotwari areas. Select the correct code a) Only 1 b) 1 and 2 c) 1 and 3 d) 2 and 3

Correct Answer: A Explanation Solution (a) To carry forward the mandate given by the Lahore Congress, Gandhi presented eleven demands to the Government and gave an ultimatum of January 31, 1930 to accept or reject these demands . The demands were as follows. Issues of General Interest 1. Reduce expenditure on Army and civil services by 50 per cent. 2. Introduce total prohibition. 3. Carry out reforms in Criminal Investigation Department (CID). National Movement 1919-1939 167 4. Change Arms Act allowing popular control of issue of firearms licences. 5. Release political prisoners. 6. Accept Postal Reservation Bill. Specific Bourgeois Demands 7. Reduce rupee-sterling exchange ratio to is 4d 8. Introduce textile protection. 9. Reserve coastal shipping for Indians. Specific Peasant Demands 10. Reduce land revenue by 50 per cent. 11. Abolish salt tax and government's salt monopoly. Defiance of the salt laws started all over the country. In Tamil Nadu, C. Rajagopalachari led a march from Tiruchirapally to Vedaranniyam. In Malabar, K. Kelappan led a march from Calicut to Poyannur. In Assam, satyagrahis walked from Sylhet to Noakhali (Bengal) to make salt. In Andhra, a number of sibirams (camps) came up in different districts as headquarters of salt satyagraha. Gandhi's arrest came on May 4, 1930 when he had announced that he would lead a raid on Dharsana Salt Works on the west coast. Gandhi's arrest was followed by massive protests in Bombay, Delhi, Calcutta and in Sholapur, where the response was the fiercest.After Gandhi's arrest, the CWC sanctioned: • Non-payment of revenue in Ryotwari areas; • No chowkidara tax campaign in zamindari areas ; • Violation of forest laws in the Central Provinces. Do you know? · Gandhi started Dandi March with 78 members of Sabarmati Ashram. · After Gandhi was arrested, Sarojini Naidu, Imam Sahib and Manila (Gandhi's son) took up the unfinished task of leading a raid on Dharsana Salt Works. THINK! · Why salt was chosen as a central formula for Civil disobedience movement? · Methods of protest used in the movement

QUESTION 8. Consider the following Paper/Journal Author 1. Bandi Jeevan : Sachin Sanyal 2. The Philosophy of : Bhagat Singh the bomb 3. Pather Dabi : J.M. Sengupta Which of the pairs given above is/are correctly matched? a) Only 1 b) Only 2 c) 1 and 2 d) 2 and 3

Correct Answer: A Explanation Solution (a) · Bandi Jeevan: Sachin Sanyal · The Philosophy of bomb: Bhagwati Charan Vohra · Pather Dabi: Saratchandra Chatterjee THINK! · Important papers and journals written during freedom struggle

QUESTION 7. Which among the following sections opposed council entry during passive phase of movement in 1922? a) Swarajists b) No-Changers c) Responsivists d) Non-responsivists

Correct Answer: B Explanation Solution (b) After Gandhi's arrest (March 1922), there was disintegration, disorganisation and demoralisation among nationalist ranks. A debate started among Congressmen on what to do during the transition period, i.e., the passive phase of the movement. Those advocating entry into legislative councils came be known as the Swarajists led by C.R. Das, Motilal Nehru and Ajmal Khan. The other school of thought led Vallabhbhai Patel, Rajendra Prasad, C. Rajagopalachari and M.A. Ansari came to be known as the 'No-changers'. The 'No-changers' opposed council entry, advocated, concentration on constructive work, and continuation of boycott noncooperation, and quiet preparation for resumption of the suspended civil disobedience programme. By 1924, the Swarajist position had weakened because of widespread communal riots; split among Swarajists themselves on communal and Responsivist-Non-responsivist lines, and the death of C.R. Das in 1925 weakened it further. The Responsivists among Swarajists—Lala Lajpat Rai, Madan Mohan Malaviya and N.C. Kelkar—advocated cooperation with the Government and holding of office wherever possible to protect the so called Hindu interests. They accused the Non-responsivists like Motilal Nehru of being anti-Hindu and a beef-eater. Do you know? · Due to differences between the two schools and defeat of the Swarajists' proposal of 'ending or mending' the councils at the Gaya session of the Congress (December 1922), C.R Das and Motilal Nehru resigned from the Congress and announced formation of Congress-Khilafat Swarajya Party. · However, both sides wanted to avoid a split like 1907 and hence reached a compromise where the Swarajists were allowed to contest elections as a group within the Congress. THINK! · Surat split · Analysis of work done by Swarajista and no-changers

QUESTION 96. 'World Inequality Report- 2018' is published by a) World Bank b) International Monetary Fund c) World Inequality Lab d) United Nations Development Programme

Correct Answer: C Explanation Solution (c) Deregulation and opening-up reforms in India since 1980s have led to substantial increase in inequality so much that top 0.1% of earners has continued to capture more growth than all those in the bottom 50% combined, said the World Inequality Lab in its World Inequality Report 2018 http://www.thehindu.com/opinion/op-ed/focus-on-income-mobility/article21859319.ece

QUESTION 9. Which of the following statements are correct? 1. Bhagat Singh, Sukhdev and Rajguru were tried in Meerut Conspiracy case. 2. Surya sen and his associated conducted Chittagong armoury raid. 3. Yugantar group was a secret revolutionary association operating in Bengal. Select the correct code a) 1 and 2 b) 1 and 3 c) 2 and 3 d) 1,2 and 3

Correct Answer: C Explanation Solution (c) Lahore conspiracy case : Bhagat Singh, Azad and Rajguru shot dead Saunders, the police official responsible for the lathicharge in Lahore in December 1928. Later, Bhagat Singh and Batukeshwar Dutt were asked to throw a bomb in the Central Legislative Assembly on April 8, 1929 against the passage of the Public Safety and Trade Disputes Bill aimed at curtailing civil liberties of citizens in general and workers in particular. Bhagat Singh was caught during this; however, Bhagat Singh, Sukhdev and Rajguru were tried in the Lahore conspiracy case. The Meerut Conspiracy Case was a controversial court case initiated in British India in March 1929 and decided in 1933. Several trade unionists, including three Englishmen were arrested for organizing an Indian railway strike. The British Government convicted 33 leftist trade union leaders under a false law suit. The Meerut Conspiracy case trial helped the Communist Party of India to consolidate its position among workers. Chittagong Armoury Raid (April 1930): Surya Sen decided to organise an armed rebellion along with his associates—Anant Singh, Gariesh Ghosh and Lokenath Baul to show that it was possible to challenge the armed might of the mighty British Empire. They had planned to occupy two main armouries in Chittagong to seize and supply arms to the revolutionaries to destroy telephone and telegraph lines and to dislocate the railway link of Chittagong with the rest of Bengal. The raid was conducted in April 1930 and involved 65 activists under the banner of Indian Republican Army— Chittagong Branch. The raid was quite successful; Sen hoisted the national flag, took salute and proclaimed a provisional revolutionary government. Jugantar or Yugantar was one of the two main secret revolutionary trends operating in Bengal for Indian Independence . Leaders like Aurobindo Ghosh, his brother Barin ghosh, Bhupendranath dutta and Raja Subodh Malik established this party in April 1906 Do you know? · Jatin Das became the first martyr on the 64th day of his fast. · There was a large-scale participation of young women in revolutionary activities especially under Surya Sen like Pritilata Waddedar and Kalpana Dutt. THINK! · Revolutionary activities and major leaders

QUESTION 58. Who of the following received 'Kaisar - I - Hind' for supporting British during World War - I? a) Rabindranth Tagore b) Lala Lajpat Rai c) Mahatma Gandhi d) Madan Mohan Malviya

Correct Answer: C Explanation Solution (c) • The Kaisar-i-Hind Medal for Public Service in India was a medal awarded by the British monarch to civilians of any nationality who rendered distinguished service in the advancement of the interests of the British Raj. • Mahatma Gandhi was awarded the Kaisar-i-Hind in 1915 by The Lord Hardinge of Penshurst for his support to British during World War I. He urged Indians to support British at the time of war and also asked youth to join army. Do you know? · Gandhi returned the medal in 1920 as part of the national campaign protesting the Jallianwala Bagh massacre

QUESTION 56. Consider the following statements regarding the approach of Extremists in Indian National Congress: 1. It believed in complete independence of India from British rule. 2. They were revolutionary in approach and believed in armed rebellion. 3. They took ideological inspiration from Indian history and Hindu cultural symbols. Which of the above statements is/are NOT correct? a) 1 and 2 b) 3 only c) 1 and 3 d) 2 and 3

Correct Answer: A Explanation Solution (a) Note: Incorrect options have been asked. There was some ambivalence regarding the goal of Extremist leaders, although they spoke of it as 'swaraj'. Tilak famously spoke of swaraj as his birthright, but never clearly stated whether he meant a complete rejection of and breach with the British. Lajpat Rai emphasized more the need for a social revolution within native society in the struggle against the British. Bipin Chandra Pal spoke of political autonomy at all costs. They demanded equal rights and liberty for Indians not as British subjects, but because these were fundamental human rights as enunciated by the French Revolution. The main methods of agitation preached by Extremists were swadeshi and boycott. Swadeshi entailed positive and active encouragement and patronage to indigenous products and production techniques, and boycott was a negative programme of rejecting British goods, and by extension, British institutions like schools and colleges, law courts, administrative and legislative posts. The earliest Extremist politics envisaged a programme of 'constructive swadeshi', which entailed setting up swadeshi enterprises and stores, the spread of vernacular education and social work in villages, and the use of traditional and customary festivals to draw in common people. Rabindranath Tagore was an important proponent of this approach. By 1906, a new radicalism infused extremist politics, as extended boycott and strikes were undertaken through mass participation in the agitation against the partition of Bengal, and complete swaraj was established as the objective. Think · Difference between the approach of extremists and revolutionaries

QUESTION 19. Consider the following statements w.r.t Quit India Movement 1. Mahatma Gandhi was named the leader of the struggle. 2. Failure of Cripps Mission was one of the reasons for launch of movement. 3. Women revolutionaries like Usha sharma and Pritilata Wadddedar took part in underground activities. 4. Parallel governments were established at Talchar, Satara and Tamluk. Select the correct code a) 1,2 and 3 b) 1,2 and 4 c) 2,3 and 4 d) 1,2,3 and 4

Correct Answer: B Explanation Solution (b) QUIT INDIA MOVEMENT The reasons for launch: 1. The failure of the Cripps Mission to solve the constitutional deadlock. 2. There was popular discontent because of rising prices and shortage of rice, salt, etc. 3. News of reverses' suffered by the British in South-East Asia. 4. The manner in which the British evacuated from South-East Asia leaving the subjects to their fate. 5. The leadership wanted to condition the masses for a possible Japanese invasion. Gandhi was named the leader of the struggle, but after his arrest there was a leadership vacuum and thus it is often called 'leaderless movement'. Rammanohar Lohia, Jayaprakash Narayan, Aruna Asaf Ali, Usha Sharma, Biju Patnaik, Chhotubhai Puranik, Achyut Patwardhan, Sucheta Kripalani and R.P. Goenka carried out underground activity. Usha Sharma started an underground radio in Bombay . Pritilata Waddedar was an Indian Bengali revolutionary and part of revolutionary group headed by Surya Sen . She led a team of fifteen revolutionaries in a 1932 attack on the Pahartali European Club, which had a sign board that read "Dogs and Indians not allowed".The revolutionaries torched the club and were later caught by the British police. To avoid getting arrested, Pritilata consumed cyanide and died. Parallel governments were established in Ballia(UP), Tamluk(Bengal), Satara (Maharashtra) and Talchar(Orissa). Do you know? · First parallel government was established at Ballia and the longest one was at Satara. · The movement is also known as 'August Kranti'. THINK! · Important facts about Quit India Movement

QUESTION 17. Arrange the following in chronological order 1. Wavell Plan 2. Cripps Mission 3. Formation of Indian National Army 4. CR Formula Select the correct code a) 2-3-1-4 b) 3-2-1-4 c) 2-3-4-1 d) 3-2-4-1

Correct Answer: C Explanation Solution (c) · Wavell Plan: May 1945 · Cripps Mission: March 1942 · Formation of Indian National Army: August 1942 · CR Formula: 1944 THINK! · Indian National Army - Formation and its role · CR Formula · Wavell Plan

QUESTION 80. Which of the following reasons were responsible for the decline of Indian Home Rule League movement? 1. The movement was left leaderless once Tilak moved to England. 2. The rise of mahatma Gandhi and his new way of revolution was like by the masses 3. Montague - Chelmsford reforms were announced by the Government Select the code from following: a) 1 and 2 b) 2 and 3 c) 3 only d) All of the above

Correct Answer: D Explanation Solution (d) The following reasons were responsible for the Decline of Home Rule League in India: The Movement was left leaderless once Tilak left for England to pursue a libel case he had filed and Annie Besant was largely satisfied by the promise of Reforms. Its further growth and activity were stalled by the rise of Mahatma Gandhi and his Satyagraha a rt of revolution: non-violent, but mass-based civil disobedience. Gandhi's Hindu lifestyle, mannerisms and immense respect for Indian culture and the common people of India made him immensely popular with India's common people. His victories in leading the farmers of Champaran, Bihar and Kheda, Gujarat against the British authorities on tax revolts made him a national hero. After the Montagu Declaration the league agreed to suspend its expansion of the movement. After this the all moderate candidate gave up the membership of league. The league believed that the British government will gradually reform the administration and local representative system by ushering participation of local Indians. Think · Compare the Home Rule League movement started by Annie Besant and Tilak

QUESTION 76. Who was the first Indian to be appointed for the Viceroy's executive Council? a) Dada Bhai Naoroji b) Pherozshah Mehta c) Satyendra Sinha d) Abhindranath Tagore Correct Answer: C

Explanation Solution (c) The Indian Councils Act 1909 empowered the Governor General to nominate one Indian member to the Executive Council leading to the appointment of Satyendra Prasanno Sinha as the first Indian member. The Government of India Act 1919 increased the number of Indians in the council to three. Think · From which Act the Executive council of Viceroy was formed? · When was a Law member added to the Executive Council?

QUESTION 59. Which of the following statements are correct regarding the formation of All India Muslim League? 1. It was established in the year 1906 2. It was formed at Dhaka and had headquarter at Lucknow 3. It was established as an opposition party to the congress after split of congress as extremist and moderates. Select the code from following: a) 1 and 2 b) 2 and 3 c) 1 and 3 d) All of the above

Correct Answer: A Explanation Solution (a) A resolution to form the All India Muslim League was passed by Nawab Salimullah Khan and was seconded by Hakim Ajmal Khan, Maulana Muhammad Ali and Moulana Zafar Ali. The resolution was passed by All India Educational Conference on 30th December 1906. A committee was formed to prepare its draft constitution. Sir Agha Khan was appointed as President and Syed Hassan Balgrami was appointed as secretary, while Nawab Mohsim-ul-Mulk and Nawab Viqar-ul-Mulk were made joint secretaries with six Vice- Presidents, a Central Committee with forty Members was also constituted. In this way Muslim league was established and become the sole representative of Muslims. Knowing the circumstances which led to the formation of Muslim league was not difficult to make out what it aimed to. However, the Muslim league laid the following points as its objectives. 1. To create among Muslims the feelings of loyalty towards British Government and to remove misconception and suspicious. 2. To Safeguard the political rights of the Muslims and to bring them into the notice of the Government. 3. To prevent among the Muslims, the rise of prejudicial feelings against the other communities of India. The first session of all India Muslim league was held at Karachi on 29th December, 1907 and was presided over by Adamji Peer Bhai. Think · Was Muslim League against Congress in its initial phase?

QUESTION 10. Consider the following statements 1. Simon commission was appointed by Lord Birkenhead to recommend constitutional reforms for India. 2. Hindu Mahasabha and Justice Party in the south did not boycott the commission. Which of the above statement/s is/are correct? a) Only 1 b) Only 2 c) 1 and 2 d) None

Correct Answer: A Explanation Solution (a) ANTI-SIMON COMMISSION UPSURGE There was a chorus of protest by all Indians against the appointment of an all-white, seven- member Indian Statutory Commission, popularly known as the Simon Commission (after the name of its chairman Sir John Simon), on November 8, 1927. The commission was to recommend to the Government whether India was ready for further constitutional reforms and on what lines. (Although constitutional reforms were due only in 1929, the Conservative Government, then in power in Britain, feared defeat by the Labour Party and did not want to leave the question of the future of Britain's most priced colony in "irresponsible Labour hands".) The Conservative Secretary of State, Lord Birkenhead, who had constantly talked of the inability of Indians to formulate a concrete scheme of constitutional reforms, which had support of wide sections of Indian political opinion, was responsible for the appointment of the Simon Commission. The Congress session in Madras (December 1927) meeting under the presidency of M.A. Ansaii decided to boycott the commission "at every stage and in every form". Those who decided to support the Congress call of boycott included the Liberals of the Hindu Mahasabha and the majority faction of the Muslim League under Jinnah. Some others, such as the Unionists in Punjab and the Justice Party in the south, decided not to boycott the commission. Do you know? · Lala Lajpat Rai died due to lathi charge during protests against Simon Commission. · Bhagat singh later killed the police official Saunders responsible for lathi charge and was tried and hanged for this. THINK! · Impact of simon commission

QUESTION 63. Who of the following personalities were called 'The father of Indian Unrest'? a) Mahatma Gandhi b) Mohammad Ali Jinnah c) Praful Chaki d) Bal Gangadhar Tilak

Correct Answer: D Explanation Solution (d) Valentile Chirol has called Bal Gangadhar Tilak as father of Indian Unrest. Think · Grand old Man of South India · Father of Modern India

QUESTION 2. Which of the following are features of Government of India Act, 1919? 1. Ministers had to resign if a no-confidence motion was passed against them. 2. Women were given the right to vote. 3. Subjects were divided into 'reserved' and 'transferred' lists. Select the correct code a) 1 and 2 b) 1 and 3 c) 2 and 3 d) 1,2 and 3

Correct Answer: D Explanation Solution (d) The main features of the Government of India Act, 1919 were as follows: · Dyarchy, i.e., rule of two—executive councilors and popular ministers was introduced. · Subjects were divided into two lists: "reserved" which included subjects such as law and order, finance, land revenue, irrigation, etc., and "transferred" subjects such as education, health, local government, industry, agriculture, excise, etc. · The ministers were to be responsible to the legislature and had to resign if a no-confidence motion was passed against them by the legislature, while the executive councillors were not to be responsible to the legislature. · Provincial Legislative Councils were further expanded-70% of the members were to be elected. · The system of communal and class electorates was further consolidated. · Women were also given the right to vote. · The Legislative Councils could initiate legislation but the governor's assent was required. The governor could veto bills and issue ordinances. · The Legislative Councils could reject the budget but the governor could restore it, if necessary. · The legislators could ask questions and supplementaries, pass adjournment motions and vote a part of the budget, but 75% of the budget was still not votable. Do you know? · Allocation of seats for Central Legislature to provinces was based on 'importance' of provinces —for instance, Punjab's military importance, and Bombay's commercial importance. · The secretary state was henceforth to be paid out of the' British exchequer. THINK! · Impact of Government of India Act, 1919

QUESTION 100. A Lagrange point is a location in space where the combined gravitational forces of two large bodies, such as Earth and the sun or Earth and the moon, equal the centrifugal force felt by a much smaller third body. Select the correct statement about 'L1 point'? a) L1, lies between Earth and the Sun at about 1 million miles from Earth b) L1, lies a million miles from Earth, but in the opposite direction of the Sun c) L1, lies behind the Sun, opposite Earth's orbit. d) L1, lies between Earth and the Moon at about 1 million miles from Earth

Correct Answer: A Explanation Solution (a) A Lagrange point is a location in space where the combined gravitational forces of two large bodies, such as Earth and the sun or Earth and the moon, equal the centrifugal force felt by a much smaller third body. The interaction of the forces creates a point of equilibrium where a spacecraft may be "parked" to make observations. These points are named after Joseph-Louis Lagrange, an 18th-century mathematician who wrote about them in a 1772 paper concerning what he called the "three-body problem." They are also called Lagrangian points and libration points. There are five Lagrange points around major bodies such as a planet or a star. Three of them lie along the line connecting the two large bodies. In the Earth-sun system, for example, the first point, L1, lies between Earth and the sun at about 1 million miles from Earth. L1 gets an uninterrupted view of the sun, and is currently occupied by the Solar and Heliospheric Observatory (SOHO) and the Deep Space Climate Observatory. L2 also lies a million miles from Earth, but in the opposite direction of the sun. At this point, with the Earth, moon and sun behind it, a spacecraft can get a clear view of deep space. NASA's Wilkinson Microwave Anisotropy Probe (WMAP) is currently at this spot measuring the cosmic background radiation left over from the Big Bang. The James Webb Space Telescope will move into this region in 2018. The third Lagrange point, L3, lies behind the sun, opposite Earth's orbit. For now, science has not found a use for this spot, although science fiction has. Points L4 and L5, however, are stable, "like a ball in a large bowl," according to the European Space Agency. These points lie along Earth's orbit at 60 degrees ahead of and behind Earth, forming the apex of two equilateral triangles that have the large masses (Earth and the sun, for example) as their vertices. http://www.thehindu.com/sci-tech/science/workings-of-solar-wind-flows-deciphered-by-prl-team/ article21380124.ece

QUESTION 5. Which of the following are true about Nagpur Session of Indian National Congress in 1920? 1. Provincial congress committees were organized on linguistic basis. 2. Programme of non-cooperation was endorsed. 3. First All India Youth Congress was formed. Select the correct code a) 1 and 2 b) 1 and 3 c) 2 and 3 d) 1,2 and 3

Correct Answer: A Explanation Solution (a) At the Nagpur session of the Indian National Congress in December 1920— · The programme of non-cooperation was endorsed ; · The Congress decided to have the attainment of swaraj through peaceful and legitimate means as its goal; · Some important organisational changes were made: a Congress Working Committee (CWC) of 15 members was set up to lead the Congress from now onwards; Provincial Congress Committees on linguistic basis were organised ; ward committees was organised; and entry fee was reduced to four arenas; · Gandhi declared that if the non-cooperation programme was implemented completely, swaraj would be ushered in within a year. · All India Youth Congress was formed through Calcutta Session of 1928. Do you know? · At this stage, some leaders like Mohammad Ali Jinnah, Annie Besant, G.S. Kharpade and B.C. Pal left the Congress as they believed in a constitutional and lawful struggle. THINK! · Important Congress sessions

QUESTION 24. Consider the following statements about Cripps Mission 1. It was sent due to pressure from Allies to seek Indian co-operation during the war. 2. It offered right to secede to provinces. 3. Constitution making was to be 'solely' in Indian hands unlike the August offer where it was 'mainly' in Indian hands. 4. Jawahar Lal Nehru and Sardar Patel were the official negotiators for the Congress. Which of the statements give above are correct? a) 1,2 and 3 b) 1,3 and 4 c) 1,2 and 4 d) 1,2,3 and 4

Correct Answer: A Explanation Solution (a) CRIPPS MISSION In March 1942, a mission headed by Stafford Cripps was sent to India due to the following reasons: · Reverses suffered by Britain in South-East Asia, the Japanese threat to invade India seemed real now and Indian support became crucial. · There was pressure on Britain from the Allies (USA, USSR, China) to seek Indian cooperation . · Indian nationalists had agreed to support the Allied cause if substantial power was transferred immediately and complete independence given after the war. The main proposals of the mission were as follows. 1. An Indian Union with a dominion status would be set up 2. After the end of the war, a constituent assembly would be convened to frame a new constitution. Members of this assembly would be partly elected by the provincial assemblies through proportional representation and partly nominated by the princes. 3. The British Government would accept the new constitution subject totwo conditions: (i) Any province not willing to join the Union could have a separate constitution and form a separate Union , (ii) The new constitution making body and the British Government would negotiate a treaty to effect the transfer of power and to safeguard racial and religious minorities. 4. In the meantime, defence of India would remain in British hands and the governor-general's powers would remain intact. The proposals differed from those offered in the past in many respects— The making of the constitution was to be solely in Indian hands now (and not "mainly" in Indian hands—as contained in the August Offer). A concrete plan was provided for the constituent, assembly. · Option was available to any province to have a separate constitution—a blueprint for India's partition. · Free India could withdraw from the Commonwealth. · Indians were allowed a large share in the administration in the interim period. Nehru and Maulana Azad were the official negotiators for the Congress . Do you know? · Both Congress and the Muslim League rejected the proposal. THINK! · Reasons for failure of Cripps Mission

QUESTION 75. Which of the following statement is correct regarding 'Shimla Deputation'? a) A group of Muslim leaders met Lord Minto to ask for Separate electorates for Muslims. b) Leaders from Congress met Lord Minto to annul separation of Bengal c) A commission was sent by British Government from England to meet Indian leaders and examine the implementation of Indian Councils Act 1892. d) None of the above

Correct Answer: A Explanation Solution (a) On 8 October 1906 a deputation of prominent Muslims led by the Aga Khan visited Viceroy Minto at Simla to present their demands. The demands were set out in what has become known as 'The Simla Deputation'. In they demanded that in all local and provincial elections Muslims should have their own representatives and that Muslims in all councils should have ahigher percentage of seats than their percentage of the Indian population. The British acceptance of the Simla Deputation was an extremely important moment in the history of the subcontinent. The acceptance showed that the attempts made by Sir Syed and others to restore relations between the Muslims and the British had been successful. Think · When was the concept of separate electorate started in India? · Who else were given separate electorate in 1909 apart from Muslims?

QUESTION 77. The Government of India passed 'Defence of India Act' in 1915. Which of the following statements is correct regarding this Act? a) It was an emergency criminal law enacted to check nationalist and revolutionary activity during and in the aftermath of World War I b) The law was passed to strengthen the Indian army to prepare for another emergency like World War I c) The Act was passed to officially make India a party in World War I d) None of the above

Correct Answer: A Explanation Solution (a) Defence of India Act - 1915 The Defence of India Act 1915 , also referred to as the Defence of India Regulations Act , was an emergency criminal law enacted by the Governor-General of India in 1915 with the intention of curtailing the nationalist and revolutionary activities during and in the aftermath of the First World War. It was similar to the British Defence of the Realm Acts, and granted the Executive very wide powers of preventive detention, internment without trial, restriction of writing, speech, and of movement. However, unlike the English law which was limited to persons of hostile associations or origin, the Defence of India act could be applied to any subject of the King, and was used to an overwhelming extent against Indians. The passage of the act was supported unanimously by the non-official Indian members in the Viceroy's legislative council, and was seen as necessary to protect against British India from subversive nationalist violence. The act was first applied during the First Lahore Conspiracy trial in the aftermath of the failed Ghadar Conspiracy of 1915, and was instrumental in crushing the Ghadr movement in Punjab and the Anushilan Samiti in Bengal. However its widespread and indiscriminate use in stifling genuine political discourse made it deeply unpopular, and became increasingly reviled within India. The extension of the law in the form of the Rowlatt Act after the end of World War I was opposed unanimously by the non-official Indian members of the Viceroy's council. It became a flashpoint of political discontent and nationalist agitation, culminating in the Rowlatt Satyagraha. The act was re-enacted during World War II as Defence of India act 1939. Independent India retained the law in a number of amended forms, which have seen use in proclaimed states of national emergency including Sino-Indian War, Bangladesh crisis, The Emergency of 1975 and subsequently the Punjab insurgency. Think · Rowlatt Act · Rowlatt Act Satyagraha

QUESTION 61. Consider the following statements regarding Emperor vs Aurobindo Ghosh and others case, or famously known as Alipore Conspiracy case: 1. The case was held for attempt to murder Presidency magistrate Kingsford by a bomb blast. 2. As chief conspirator Aurobindo Ghosh was given a death sentence Which of the above statements is/are correct? a) 1 only b) 2 only c) Both 1 and 2 d) Neither 1 nor 2

Correct Answer: A Explanation Solution (a) Emperor vs Aurobindo Ghosh and others , colloquially referred to as the Alipore Bomb Case , the Muraripukur conspiracy , or the Manicktolla bomb conspiracy , was a criminal case held in India in 1908. The case saw the trial of a number of Indian nationalists of the Anus hilan Samiti in Calcutta, under charges of "Waging war against the Government" of the British Raj. The trial was held at Alipore Sessions Court, Calcutta, between May 1908 and May 1909. The trial followed in the wake of the attempt on the life of Presidency Magistrate Douglas Kingsford in Muzaffarpur by Bengali nationalists Khudiram Bose and Prafulla Chaki in April 1908, which was linked to the attempts to derail the train carrying Lieutenant-Governor Sir Andrew Fraser in December 1907. Among the famous accused were Aurobindo Ghosh, his brother Barin Ghosh as well as 37 other Bengali nationalists of the Anushilan Samiti . Most of the accused were arrested from Barin Ghosh's Garden house in 36 Murarirupukur Road, in the Manicktolla suburb of Calcutta. The accused were held in the Presidency Jail in Alipore before the trial, where Narendranath Goswami, approver and crown-witness, was shot dead by two fellow accused Kanailal Dutta and Satyendranath Bose within the jail premises. Goswami's murder led to collapse of the case against Aurobindo. However, his brother Barin and a number of others were convicted of the charges and faced varying jail terms from life-imprisonment to shorter jail terms. Aurobindo Ghosh retired from active nationalist politics after serving a prison sentence awarded in the trial, beginning his journey into spirituality and philosophy that he described as having started with revelations that occurred to him during his incarceration. He later moved to Pondicherry, establishing an Ashram . For Anushilan samiti , the incarceration of many of its prominent leaders led to a decline in the influence and activity of the Manicktolla branch, and its activities were overtaken by what emerged to be called the Jugantar branch under the leadership of Bagha Jatin. Think · Aurobindo ashram

QUESTION 18. Which of the following events took place during the tenure of Viceroy Lord Irwin? 1. Visit of Simon commission to India 2. Dandi March by Gandhi 3. Boycott of First Round Table Conference 4. Poona Pact Select the correct code a) 1,2 and 3 b) 1,3 and 4 c) 2,3 and 4 d) 1,2,3 and 4

Correct Answer: A Explanation Solution (a) Lord Irwin (1926-1931) · Visit of Simon Commission to India (1928) and the boycott of the commission by the Indians. · An All-Parties Conference held at Lucknow (1928) for suggestions for the (future) Constitution of India, the report of which was called the Nehru Report or the Nehru Constitution. · Appointment of the Harcourt Butler Indian States Commission (1927). · Murder of Saunders, the assistant superintendent of police of Lahore; bomb blast in the Assembly Hall of Delhi (1929); the Lahore Conspiracy Case and death of Jatin Das after prolonged hunger strike (1929), and bomb accident in train in Delhi (1929). · Lahore session of the Congress (1929); Purna Swaraj Resolution. · Dandi March (March 12, 1930) by Gandhi to launch the Civil Disobedience Movement. · 'Deepavali Declaration' by Lord Irwin (1929). · Boycott of the First Round Table Conference (1930), Gandhi-Irwin Pact (1931) and suspension of Civil Disobedience Movement. Poona Pact was signed in 1932 under Lord Willingdon. Do you know? · Civil Disobedience movement was suspended after Gandhi-Irwin pact, but resumed after failure of second RTC. THINK! · Major events under Lord Willingdon

QUESTION 52. 'New Lamps for Old' was a series of articles written by Aurobindo Ghosh. What was the content of these articles? a) It was a fiery critique of modern politics by Congress. b) It was a book on applicability of Hindu philosophy in modern times c) It was a critique on monotheistic religions of the world d) None of the above

Correct Answer: A Explanation Solution (a) New Lamps for Old Aurobindo Ghosh gave a systematic critique of the moderate politics of Indian National Congress in series of articles entitled NEW LAMPS FOR OLD. They were begun at the instance of K. G. Deshpande, Aurobindo's Cambridge friend who was editor of the paper, but the first two articles made a sensation and frightened Ranade and other Congress leaders. Ranade warned the proprietor of the paper that, if this went on, he would surely be prosecuted for sedition. Accordingly the original plan of the series had to be dropped at the proprietor's instance. Deshpande requested Sri Aurobindo to continue in a modified tone and he reluctantly consented, but felt no farther interest and the articles were published at long intervals and finally dropped of them altogether. Think · Anushilan Samiti · Aurobindo Ashram

QUESTION 70. Who was the first Women president of Indian National Congress? a) Sarojini Naidu b) Annie Besant c) Indira Gandhi d) Lakshmi Sehgal

Correct Answer: B Explanation Solution (b) Annie Besant was the first women president of Congress. she presided the congress session of 1917. She played a key role to bring moderates and extremist on the single platform once again. Sarijini Naidu was the first Indian Women President of INC - 1925. Think · How many women presidents have been there in Indian National Congress before independence?

QUESTION 4. Which of the following statements w.r.t. Nehru Report are correct? 1. It was the first major attempt by the Indians to draft a constitutional framework for the country. 2. It demanded 'complete independence'. 3. Independence for India League was founded in opposition to the Nehru Report. Select the correct code a) 1 and 2 b) 1 and 3 c) 2 and 3 d) 1,2 and 3

Correct Answer: B Explanation Solution (b) As an answer to Lord Birkenhead's challenge, an All Parties Conference met in February 1928 and appointed a subcommittee under the chairmanship of Motilal Nehru to draft a constitution. This was the first major attempt by the Indians to draft a constitutional framework for the country . The committee included Tej Bahadur Sapru, Subhash Bose, M.S. Aney, Mangal Singh, Ali Imam, Shuab Qureshi and G.R. Pradhan as its members. The report was finalised by August 1928. Main Recommendations 1 . Dominion status on lines of self-governing dominions as the form of government . 2. Rejection of separate electorates; instead, a demand for joint electorates with reservation of seats for Muslims. 3. Linguistic provinces. 4. Nineteen fundamental rights including equal rights for women, right to form unions, and universal adult suffrage. 5. Responsible government at the centre and in provinces 6. Full protection to cultural and religious interests of Muslims. 7. Complete dissociation of state from religion. Not only were the Muslim League, the Hindu Mahasabha and the Sikh communalists unhappy about the Nehru Report, but the younger section of the Congress led by Jawaharlal Nehru and Subhash Bose were also angered. The younger section regarded the idea of dominion status in the report as a step backward, and the developments at the All Parties Conference strengthened their criticism of the dominion status idea. Nehru and Subhash Bose rejected the Congress' modified goal and jointly set up the Independence for India League . Do you know? · Jinnah's fourteen points came as a response to Nehru Report and became the basis for all future propaganda of the Muslim League. THINK! · Delhi proposals · Responses to Nehru Report

QUESTION 28. Consider the following statements w.r.t revolt of Royal Indian Navy ratings 1. Sardar Patel asked the ratings to surrender before the authority. 2. Congress showed indifference to the revolutionary situation. 3. M.A. Jinnah persuaded the ratings to continue strike until their demands are met. Which of the above given statements are true? a) 1 and 2 b) 1 and 3 c) 2 and 3 d) 1,2 and 3

Correct Answer: A Explanation Solution (a) On February 18, 1946, 1100 naval Ratings of HMIS Talwar went on a strike to protest against: · Racial discrimination (demanding equal pay for Indian and white soldiers) · Unpalatable food · Abuse by superior officers · Arrest of a rating for scrawling. 'Quit India' on HMIS Talwar · INA trials · Use of Indian troops in Indonesia, demanding their withdrawal. There were sympathetic strikes in military establishments in Karachi, Madras, Visakhapatnam, Calcutta, Delhi, Cochin, Jamnagar, Andamans, Bahrain and Aden. Also, there were strikes by the Royal Indian Air Force in Bombay, Poona, Calcutta, Jessore and Arnbala. Patel and Jinnah persuaded the ratings to surrender on February 23 with an assurance that national parties would prevent any victimisation . The leftists claim that the Congress indifference to the revolutionary situation arose be cause of two considerations—that the situation would go out of its control and that disciplined armed forces were vital in a free India. Do you know? · Gandhi remarked that the mutiny was badly advised. · It was a Maratha battalion in Bombay that rounded up the ratings and restored them to their barracks. THINK! INA trials

QUESTION 44. International Workshop on Physics of Semiconductor devices: IWPSD 2017 has been organized by 1. Solid State Physics Laboratory (SSPL), DRDO 2. IIT Delhi 3. Hindustan Aeronautics Limited Select the code from below: a) 1 and 2 b) 2 and 3 c) 1 and 3 d) All of the above

Correct Answer: A Explanation Solution (a) Organised by • Solid State Physics Laboratory (SSPL), a premier semiconductor research laboratory of DRDO • Indian Institute of Technology (IIT), Delhi In association with • Society for Semiconductor Devices • Semiconductor Society (India) • Society for Information Display Think • Modified Special Incentive Package Scheme (M-SIPS) • Electronic Development Fund (EDF) http://pib.nic.in/newsite/PrintRelease.aspx?relid=174249

QUESTION 57. Consider the following statements regarding 'Silk Letter Movement' aka 'Reshmi Rumaal Tehreek': 1. It was movement organized by Deobandi leaders from 1913 to 1920. 2. The main aim of the movement was to establish a pure Muslim empire in Punjab with the help of British. 3. The movement got its name because of the red silk handkerchiefs carried by Deobandi leaders as a mark of protest. Which of the above statements is/are correct? a) 1 only b) 2 and 3 c) 1 and 3 d) All of the above

Correct Answer: A Explanation Solution (a) Silk Letter Movement/ Reshmi Rumaal Tehreek The Silk Letter Movement refers to a movement organised by the Deobandi leaders between 1913 and 1920, aimed at freeing India from the British rule by allying with Ottoman Turkey, Imperial Germany, and Afghanistan. The plot was uncovered by Punjab CID with the capture of letters from Ubaidullah Sindhi, one of the Deobandi leaders then in Afghanistan, to Mahmud al Hasan, another leader then in Persia. The letters were written in silk cloth, hence the name. Muhammad Mian Mansoor Ansari went to Hejaz with Mahmood Hasan in September 1915. He returned to India in April 1916 with Ghalib Nama (Silk Letter) which he showed to freedom fighters in India and the autonomous area and then took it to Kabul where he reached in June 1916. With the onset of World War I, Ubaidullah Sindhi and Mehmud Hasan (principal of the Darul Uloom Deoband) had proceeded to Kabul in October 1915 with plans to initiate a Muslim insurrection in the tribal belt of India. For this purpose, Ubaid'Allah was to propose that the Amir of Afghanistan declares war against Britain while Mahmud al Hasan sought German and Turkish help. Hasan proceeded to Hijaz. Ubaid Allah, in the meantime, was able to establish friendly relations with Amir. As the plans unfolded in what came to be called the Silk Letter movement, Ubaid'Allah was able to establish friendly relations with Amir. At Kabul, Ubaid'Allah, along with some students who had preceded him to make way to Turkey to join the Caliph's " Ji had " against Britain, decided that the pan-Islamic cause was to be best served by focussing on the Indian Freedom Movement. Think · Why were silk letters used for communication?

QUESTION 53. Congress got divided into extremist and moderates in the Surat session of 1907. Which of the following statements are correct regarding Surat Session? 1. The session was presided by Rash Behari Ghosh. 2. Tilak couldn't be the President as he was from Surat. 3. Extremist formed a separate party as Congress Swaraj Party in 1907. Select the code from following: a) 1 and 2 b) 2 and 3 c) 1 and 3 d) All of the above

Correct Answer: A Explanation Solution (a) The Congress was on the brink of split. The 1907 session was to be held at Nagpur. But Nagpur was the stronghold of Bal Gangadhar Tilak. The moderates were aware that the local delegates would raise the issues in favor of the extremists. So, Gopal Krishna Gokhle got the venue changed and the new venue was Surat which was a stronghold of the Congress. • The 1907 Surat session was held at the bank of the Tapti river in Surat. • The Extremist camp was led by Lal Bal and Pal and the moderate camp was led by Gopal Krishna Gokhle. • The Surat session was presided by Dr. Rash Behari Ghosh. The congress met in an atmosphere of anger and resentment in this session. Dr. Rash Behari Ghosh was elected but the extremists had an objection to this election. Initially the extremists dominated the session but soon they accepted Dr. Rash Behari Ghosh as president and offered to cooperate. But the session was suspended. Congress got split. By the time, the next session of Congress was held in Madras in 1908 under Dr. Rash Behari Ghosh, it was the extremist camp that was facing the Lathis and arrest by the British Government who was now in its comfortable position as Congress was divided. Think · Pherozshah Mehta · Three Ps of Moderates

QUESTION 93. Select the correct statement regarding E-Way Bill under GST: a) Under GST, movement of goods costing over ₹50,000 beyond 10 km requires prior registration and generation of an e-way bill through the GST Network to track inter- and intra-State movement. b) Under GST, movement of goods costing over ₹10,000 beyond 10 km requires prior registration and generation of an e-way bill through the GST Network to track inter- and intra-State movement. c) Under GST, movement of goods costing over ₹20,000 beyond 10 km requires prior registration and generation of an e-way bill through the GST Network to track inter- and intra-State movement. d) Under GST, movement of goods costing over ₹100,000 beyond 10 km requires prior registration and generation of an e-way bill through the GST Network to track inter- and intra-State movement.

Correct Answer: A Explanation Solution (a) The E-way bill under the goods and services tax (GST) will be rolled out from January 16. The rules for implementation of nationwide e-way Bill system for inter-State movement of goods on a compulsory basis will be notified with effect from 1st February, 2018. Under GST, movement of goods costing over ₹50,000 beyond 10 km requires prior registration and generation of an e-way bill through the GST Network to track inter- and intra-State movement. http://www.financialexpress.com/economy/e-way-bill-is-it-viable-here-is-what-businesses-have- to-say/980657/

QUESTION 12. Which of the following decisions were taken at the Lahore session of Congress in 1929? 1. Boycott of Round Table Conference 2. All members of legislatures were asked to resign from their seats 3. Complete independence was declared as the aim of Congress 4. Delhi Pact was endorsed Select the correct code a) 1,2 and 3 b) 1,3 and 4 c) 2,3 and 4 d) 1,2,3 and 4

Correct Answer: A Explanation Solution (a) The following major decisions were taken at the Lahore session (December, 1929)- 1. The Round Table Conference to be boycotted; 2. Complete independence declared as the aim of the Congress; 3. CWC authorised to launch a programme of civil disobedience including non-payment of taxes and all members of legislatures asked to resign their seats; 4. January 26, 1930 fixed as the first Independence Day, to be celebrated everywhere. Gandhi- Irwin Pact is also called Delhi Pact and was signed in 1931. Do you know? · Jawaharlal Nehru was nominated the president for the Lahore session mainly due to Gandhi's backing. · The reasons to choose him were because of the appositeness of the occasion (Congress' acceptance of complete independence as its goal), and to acknowledge the upsurge of youth that had made the anti-Simon campaign a huge success. THINK! · Gandhi - Irwin Pact

QUESTION 67. In which of the following Mass movement 'Vande Mataram' became a call of agitation for Indians? a) Swadeshi movement 1905 b) Non Cooperation Movement 1920 c) Civil Disobedience Movement 1930 d) Quit India Movement 1942

Correct Answer: A Explanation Solution (a) Vande Mataram is a poem written by Bankim Chandra Chattopadhyay in 1870s, which he included in his 1881 novel Anandamath . The poem was composed into song by Rabindranath Tagore. The first two verses of the song were adopted as the National Song of India in October 1937 by Congress Working Committee prior to the end of colonial rule in August 1947. It played a vital role in the Indian independence movement, first sung in a political context by Rabindranath Tagore at the 1896 session of the Indian National Congress. It became a popular marching song for political activism and Indian freedom movement in 1905. Spiritual Indian nationalist and philosopher Sri Aurobindo referred it as "National Anthem of Bengal". The song and the novel containing it was banned by the British government, but workers and general public defied the ban, many went to colonial prisons repeatedly for singing it, and the ban was overturned by the Indians after they gained independence from the colonial rule. Vande Mataram became a call of agitation during Swadeshi Moveemnt, when partition of Bengal was announced. Think · Bankim Chandra Chatopadyay · Jana Gana Mana

QUESTION 41. Which of the following statements are correct regarding DNA fingerprinting? a) It is a method to identify an individual from a sample of DNA by analyzing its unique pattern. b) It is a method of identifying DNA by taking fingerprints of a person. c) It is a method of identifying the sex of an unknown individual by analyzing the fingerprints found at the crime scene. d) None of the above

Correct Answer: A Explanation Solution (a) • DNA fingerprinting was first developed in 1984 by Alec Jeffreys in the UK, after Jeffreys discovered that no two people could have the same DNA sequence. • DNA fingerprinting is a method used to identify an individual from a sample of DNA by looking at unique patterns in their DNA. • The uniqueness of DNA fingerprinting as a tool of investigation is not just limited to its accuracy but extends to the way it can sift through crime scene evidence. • Advanced DNA fingerprinting can make separate prints of various individuals even from a sample mixture found at the crime scene. • DNA fingerprinting is a technique that simultaneously detects lots of minisatellites in the genome to produce a pattern unique to an individual. This is a DNA fingerprint. • The probability of having two people with the same DNA fingerprint that are not identical twins is very small. Lalji Singh • He developed DNA fingerprinting for crime investigations at the Centre for Cellular and Molecular Biology (CCMB) in Hyderabad. • He is known as the 'father' of DNA fingerprinting in India. Think • DNA profiling/STR analysis

QUESTION 48. Katas Raj is a pilgrimage site of Hindus. It has a pond which is believed to have been formed by tears of Lord Shiva when he cried at the death of his wife Sati. Where is this located? a) Pakistan b) Cambodia c) Sri Lanka d) Tibet

Correct Answer: A Explanation Solution (a) • Katas Raj, also known as Qila Katas, is a Hindu pilgrimage site comprising several temples linked to one another by walkways. • The pond is named Katas (Raj temple) after Kataksha, a Sanskrit word meaning 'tearful eyes'. • The Katas Raj temple in Chakwal district of Punjab province (Pakistan) is one of the Hindu community's most well-known places of worship. • According to a legend, the pond was formed as Lord Shiva wept upon the death of wife Satti. http://www.hindustantimes.com/world-news/absence-of-lord-ram-hanuman-statues-in-katas-raj- temple-irks-pakistan-sc/story-rsegIPgqN5USBOXvXc1oXI.html

QUESTION 35. Which of the following were proposed by August offer? 1. Assurance to give complete independence to India after war. 2. Recognition of right of Indians to frame their own constitution. Select the correct code a) Only 1 b) Only 2 c) 1 and 2 d) None

Correct Answer: B Explanation Solution (b) AUGUST OFFER : Hitler's astounding success and the fall of Belgium, Holland and France put England in a conciliatory mood. To get Indian cooperation in the war effort, the viceroy announced the August Offer (August 1940), which proposed: · Dominion status as the objective for India . · Expansion of viceroy's executive council. · Setting up of a constituent assembly after the war. Indians would decide the constitution according to their social, economic and political conceptions, subject to fulfillment of the obligation of the Government regarding defence, minority rights, treaties with states, All India services. · No future constitution to be adopted without the consent of minorities. The Congress rejected the August Offer. Do you know? · August Offer recognized for the first time, the inherent right of Indians to frame their Constitution. · For the first time, Viceroy's Executive council included more Indians than whites. THINK! · Developments during World War that necessitated August Offer · Response of Indians to the offer

QUESTION 81. Which of the following Newspapers/ journals are associated with Annie Besant? 1. Young India 2. New India 3. Common Wheel Select the code from following: a) 1 and 2 b) 2 and 3 c) 1 and 3 d) All of the above

Correct Answer: B Explanation Solution (b) After making Madras her home, Annie Besant founded a weekly newspaper Commonweal in January 1914. She started a newspaper , "New India", criticized British rule and was jailed for sedition. Young India was a Journal by Mahatma Gandhi. Do you know? · A magazine with the same name, 'Young India', was also published by Lala Lajpat Rai.

QUESTION 1. Which of the following is/are correct regarding Champaran Satyagraha? 1. Peasants were forced to grow indigo on 3/10 of the total land under Tinkathia system. 2. Raj Kumar Shukla requested Mahatma Gandhi to look into the problem. 3. Gandhi used Non-Cooperation to solve the matter. Select the correct code a) Only 1 b) Only 2 c) 1 and 2 d) 2 and 3

Correct Answer: B Explanation Solution (b) Champaran Satyagraha (1917)— First Civil Disobedience Gandhi was requested by Rajkumar Shukla to look into the problems of the indigo planters, of Champaran in Bihar. The European planters had been forcing peasants to grow indigo on 3/2 0 of the total land (called tinkathia system) . When towards the end of the nineteenth century German synthetic dyes replaced indigo, the European planters demanded high rents and illegal dues from the peasants in order to maximize their profits before the peasants could shift to other crops. Besides, the peasants were forced to sell the produce at prices fixed by the Europeans. When Gandhi, joined now by Rajendra Prasad, Mazharul-Haq, Mahadeo Desai, Narhari Parekh, J.B. Kripalani, reached Champaran to probe into the matter, the authorities ordered him to leave the area at once. Gandhi defied the order and preferred to face the punishment. This passive resistance or civil disobedience of an unjust order was a novel method at that time. Finally, the authorities retreated and permitted Gandhi to make an enquiry. Now, the Government appointed a committee to go into the matter and nominated Gandhi as a member. Gandhi was able to convince the authorities that the tinkathia system should be abolished and that the peasants should be compensated for the illegal dues extracted from them. As a compromise with the planters, he agreed that only 25 per cent of the money taken should be compensated. Within a decade, the planters left the area. Gandhi had won the first battle of civil disobedience in India. Do you know? · Gandhi used civil disobedience in South Africa against Registration certificates · A new legislation in South Africa made it compulsory for Indians there to carry at all times certificates of registration with their fingerprints. THINK! · Methods of struggle employed by Gandhi Ahmedabad mill strike and Kheda Satyagraha

QUESTION 11. Which of the following statements is/are correct regarding 'Delhi Manifesto'? 1. It included demands of Muslim leaders accepted by Congress to be included in the draft constitution. 2. It asked for majority representation of Congress at the Round Table Conference. Select the correct code a) Only 1 b) Only 2 c) 1 and 2 d) None

Correct Answer: B Explanation Solution (b) Delhi Proposals: In December 1927, a large number of Muslim leaders had met at Delhi at the Muslim League session and evolved four proposals for Muslim demands to be incorporated in the draft constitution. These proposals, which were accepted by the Madras session of the Congress (December 1927), came to be known as the 'Delhi Proposals' . These were * Joint electorates in place of separate electorates with reserved seats for Muslims; * One-third representation to Muslims in Central Legislative Assembly; * Representation to Muslims in Punjab and Bengal in proportion to their population; * Formation of three new Muslim majority provinces— Sindh, Baluchistan and North-West Frontier Province. Delhi Manifesto: On November 2, 1929, a conference of prominent national leaders issued a "Delhi Manifesto" which demanded 1. That the purpose of the Round Table Conference (RTC) should be to formulate a scheme for implementation of the dominion status (thus acting as a constituent assembly) and the basic principle of dominion status should be immediately accepted; 2. That the Congress should have majority representation at the conference ; 3. Amnesty and a general policy of conciliation; Viceroy Irwin rejected these demands on December 23, 1929. The stage of confrontation was to begin now. THINK! · Nehru Report · Round Table Conferences

QUESTION 36. Consider the following statements regarding Fish production in India? 1. India is the largest producer of Fish in the world. 2. India is first in Prawn production and largest exporter of prawns in the world. Which of the above statements is/are correct? a) 1 only b) 2 only c) Both 1 and 2 d) Neither 1 nor 2

Correct Answer: B Explanation Solution (b) Fish production in India: • India is second largest fish producer in the world. • Overall fish production has increased from 0.75 million tonne of 1950-51 to 11.41 million tonne in 2016-17. • It contributes about 6.2 percent in the fish production of the world. • India achieved about 19% growth rate during 2014-17 in overall fish production. • Growth rate in marine fisheries was about 6.65 percent, whereas the growth rate of 26.07 percent has been achieved in inland fisheries in country. • India is first in prawn production and it is the largest exporter of prawns in the world too. Blue Revolution: Integrated Development and Management of Fisheries (CSS) It provides for a focused development and management of the fisheries sector to increase both fish production and fish productivity from aquaculture and fisheries resources of the inland and marine fisheries sector including deep sea fishing. Components • National Fisheries Development Board (NFDB) and its activities. • Development of Inland Fisheries and Aquaculture. • Development of Marine Fisheries, Infrastructure and Post-Harvest Operations. • Strengthening of Database & Geographical Information System of the Fisheries Sector. • Institutional Arrangement for Fisheries Sector. • Monitoring, Control and Surveillance (MCS) and other need-based Interventions. • National Scheme on Welfare of Fishermen. It is being implemented in consultation with all States & UTs. Under the scheme, it has been targeted to enhance the fish production from 107.95 lakh tonnes in 2015-16 to about 150 lakh tonnes by the end of the financial year 2019-20. Think • National Fisheries Action Plan-2020(NFAP) http://pib.nic.in/newsite/PrintRelease.aspx?relid=174223

QUESTION 88. The celebration of 70 years of India's independence is an appropriate occasion to remember women members of the Constituent Assembly formed in 1946 to debate and draft a constitution for a soon-to- be independent India. Among the 299 members, 15 were women. Consider the following 1. Hansa Mehta 2. Madam Bhika Cama 3. Vijaya Lakshmi Pandit 4. Sucheta Kripalani 5. Aruna Asaf Ali Select the correct option from above who were members of the Constituent Assembly: a) 1, 2, 4 and 5 b) 1, 3 and 4 c) 2, 3,4 and 5 d) 1, 2, 3, 4 and 5

Correct Answer: B Explanation Solution (b) In news: The contributions and role of the women who helped draft the constitution of free India was front and centre last week at the release of the Centre for Women's Development Studies' 2018 calendar - 'Women at the Midnight Hour' . The calendar revolves around two of those eminent women members of the constituent assembly - Dakshayani Velayudhan and Vijaya Lakshmi Pandit. The celebration of 70 years of India's independence is an appropriate occasion to remember women members of the Constituent Assembly formed in 1946 to debate and draft a constitution for a soon-to- be independent India. Among the 299 members, 15 were women. The y were: · Ammu Swaminathan · Annie Mascarane · Begum Aizaz Rasul · Dakshayani Velayudhan · Durgabai Deshmukh · Hansa Mehta · Kamla Chaudhri · Leela Roy · Malati Choudhury · Purnima Banerji · Raj Kumari Amrit Kaur · Renuka Ray · Sarojini Naidu · Sucheta Kripalani and · Vijaya Lakshmi Pandit.

QUESTION 98. 'Exercise Ekuverin' is a a) India and Myanmar bilateral military exercise b) India and Maldives bilateral military exercise c) India and Srilanka bilateral military exercise d) India and Japan bilateral military exercise

Correct Answer: B Explanation Solution (b) India and Maldives The focus of the eighth annual joint military training event is to enhance interoperability between the two forces for counter-terrorist operations in semi urban environment. http://www.thehindu.com/news/national/karnataka/exercise-ekuverin-in-belagavi-from-today/ article21665847.ece

QUESTION 97. India has signed loan agreement with World Bank for US$ 250 Million for SANKALP Project. What is SANKALP? a) Skills Acquire and Knowledge Awareness for Livelihood Promotion b) Skills Acquisition and Knowledge Awareness for Livelihood Promotion c) Skills Acquisition and Knowledge Awareness for Livelihood Programme d) Skills Acquisition and Knowledge Awareness for Livelihood Project

Correct Answer: B Explanation Solution (b) Indiasigned a loan agreement with World Bank under 'Skills Acquisition and Knowledge Awareness for Livelihood Promotion' World Bank will provide $250 million loan to India for skill development programme to support livelihood. The objective of the project is to enhance institutional mechanisms for skills development and increase access to quality and market-relevant training for the work force. http://pib.nic.in/newsite/PrintRelease.aspx?relid=174300 http://www.livemint.com/Politics/sI9kI35VndSWxd6Q2X2KDO/India-to-get-250-million-loan- from-World-Bank-for-skilling.html

QUESTION 3. Which of the following statements is/are correct? 1. The crowd gathered in Jallianwala Bagh in support of non-cooperation movement. 2. Rabindranath Tagore renounced his knighthood in protest against Jallianwala Bagh incident. Select the correct code a) Only 1 b) Only 2 c) Both 1 and 2 d) None of the above

Correct Answer: B Explanation Solution (b) JALLIANWALA BAGH MASSACRE (APRIL 13,1919): On Baisakhi day, a large, crowd of people mostly from neighboring villages, unaware of the prohibitory orders in the city, had gathered in this small park to protest against the arrest of their leaders, Saifuddin Kitchlew and Satyapal. The Army surrounded the gathering under orders from General Dyer and blocked the only exit point and opened fire on the unarmed crowd killing around1000. The incident was followed by uncivilized brutalities on the inhabitants of Amritsar. Rabindranath Tagore renounced his knighthood in protest. Do you know? · The Non-cooperation movement was a reaction to the oppressive policies of the British Indian government such as the Rowlatt Act and the Jallianwala Bagh massacre. THINK! · Rowlatt Act

QUESTION 60. Jugantar Patrika was a Bengali weekly newspaper. It served as a propagating organ for which of the following organizations? a) Ramakrishna Mission b) Anushilan Samiti c) Abhinav Bharat d) Hindustan Socialist Army

Correct Answer: B Explanation Solution (b) Jugantar Patrika Jugantar Patrika was a Bengali revolutionary newspaper founded in 1906 in Calcutta by Barindra Kumar Ghosh, Abhinash Bhattacharya and Bhupendranath Dutt. A political weekly, it was founded in March 1906 and served as the propaganda organ for the nascent revolutionary organisation Anushilan Samiti that was taking shape in Bengal at the time. The journal derived its name Jugantar (Lit:New Era) from a political novel of the same name by renowned Bengali author Shivnath Shastri. The journal went on to lend its name to the Western Bengal wing of the Anushilan Samiti, which came to be known as the Jugantar group. The journal expounded and justified revolutionary violence against the British Raj as a political tool for independence, and denounced the right and legitimacy of the British rule in India. It was also critical of the Indian National Congress and its moderate methods which was viewed as aiding the Raj. Its target audience was the young, literate and politically motivated youth of Bengal, and was priced at one paisa. Think · Anushilan Samiti · Early revolutionist societies in India

QUESTION 15. Consider the following statements 1. He was the first leader to be elected to the leadership of Comintern. 2. He published a journal named 'Vanguard'. 3. He was a proponent of radical humanism. Which of the following personality is best described by above given statements? a) V.D. Savarkar b) M.N. Roy c) S.A. Dange d) Subhash Chandra Bose

Correct Answer: B Explanation Solution (b) M.N. Roy- He plunged into the nationalist movement after the partition of Bengal in 1905. His early idols were the Bengali revolutionaries as well as the fiery Vinayak Damodar Savarkar. Roy left India in 1915 to acquire weapons for the fight against the British. The months he spent in the US radicalized him, and Roy improbably became one of the founders of the Mexican Communist Party. Roy rose to the highest levels of the international communist movement, before he parted ways. He returned to India only to be sent to jail for six years by the colonial government. One result of these years of incarceration was a comprehensive critique of Marxism. He founded the Communist Party of India in Tashkent in 1920. Roy was also the first to be elected to the leadership of Commintern. He was a communist leader and the editor of the communist journal Vanguard . He condemned the session court's sentence to death to 172 of the 225 accused in the Chauri Chaura incident (later, 19 were hanged and the rest transported) as against 22 policemen killed. Roy would later be the guiding light of the radical humanist movement . Among its core beliefs was that freedom is for individuals rather than collectives, social progress should be measured by the amount of freedom every individual has, human beings are innately rational, philosophy should be rooted in science rather than in religion, political change has to be preceded by a cultural renaissance and that decentralized democracy without political parties is the best way to organize political life. http://indianexpress.com/article/india/70-years-of-independence-how-communists-kept- pestering-the-british-throughout-the-freedom-struggle-4802855/ THINK! · Role of Communists in Indian freedom struggle

QUESTION 39. Scientists have proven the existence of a new form of matter which exhibits macroscopic quantum phenomenon like superconductivity, or superfluid or insulating electronic crystal. What is the name given to this matter? a) Bosonium b) excitonium c) suppermatter d) Plasma

Correct Answer: B Explanation Solution (b) Scientists have proven the existence of new form of matter called excitonium Highlights • Excitonium exhibits macroscopic quantum phenomena, like a superconductor and is made up of excitons, particles that are formed in a very strange quantum mechanical pairing. • Excitonium is a condensate—it exhibits macroscopic quantum phenomena, like a superconductor, or superfluid, or insulating electronic crystal. • When an electron, seated at the edge of the crowded-with-electrons valence band in a semiconductor, gets excited and jumps over the energy gap to the otherwise empty conduction band, it leaves behind a "hole" in the valence band. • That hole behaves as though it were a particle with positive charge, and it attracts the escaped electron. When the escaped electron with its negative charge, pairs up with the hole, the two remarkably form a composite particle, a boson - an exciton. • The team used a novel technique called momentum-resolved electron energy-loss spectroscopy (M-EELS) for the study. • The group was able to measure collective excitations of the low-energy bosonic particles, the paired electrons and holes. • The term 'excitonium' was coined in the 1960s by Harvard theoretical physicist Bert Halperin Think • Bose-Einstein condensate (BEC) http://www.thehindu.com/sci-tech/science/new-form-of-matter-excitonium-discovered/ article21379110.ece

QUESTION 84. Gita Rahasya was written by Tilak to analyse Karma Yoga which finds its reference in Geeta. Which of the following statements regarding Geeta Rahasya is/are correct? 1. It was originally written in Sanskrit. 2. Tilak wrote it while he was in prison in Mandalay, Burma. Select the code from below: a) 1 only b) 2 only c) Both 1 and 2 d) Neither 1 nor 2

Correct Answer: B Explanation Solution (b) Shrimadh Bhagvad Gita Rahasya , popularly also known as Gita Rahasya or Karmayog Shashtra , is a 1915 Marathi language book authored by Indian social reformer and independence activist Bal Gangadhar Tilak while he was in prison at Mandalay, Burma. It is the analysis of Karma yoga which finds its source in the Bhagavad Gita, the sacred book for Hindus. According to him, the real message behind the Mahabharata 's Gita is to act or perform, which is covered in the initial parts rather than renounce, which is covered in the later parts of the epic Mahabharata. He took the Mimamsa rule of interpretation as the basis of building up his thesis. Think · Karma Yoga · Mimansa Ideology

QUESTION 65. Indian Home rule Society was established in London by a) Tara Singh b) Shyamji Krishna Varma c) Lala Lajpat Rai d) Veer Savarkar

Correct Answer: B Explanation Solution (b) The Indian Home Rule Society (IHRS) was an Indian organisation founded in London in 1905 that sought to promote the cause of self-rule in British India. The organisation was founded by Shyamji Krishna Varma, with support from a number of prominent Indian nationalists in Britain at the time, including Bhikaji Cama, Dadabhai Naoroji and S.R. Rana, and was intended to be a rival organisation to the British Committee of the Indian National Congress that was the main avenue of the loyalist opinion at the time. Founded on 18 February 1905, the IHRS was a metropolitan organisation modelled after Victorian public institutions of the time. It had a written constitution and the stated aims to "secure Home Rule for India, and to carry on a genuine Indian propaganda in this country by all practicable means". The IHRS was open for membership "to Indians only", and found significant support amongst Indian students and other Indian populations in Britain. It recruited from amongst young Indian activists, collected money, and may have been collecting arms and maintaining close contact with revolutionary movements in India. The society was foundations of the India House and, along with Krishna Varma's journal The Indian Sociologist , was the foundation of the militant Indian nationalist movement in Britain. After Krishna Varma's shift to Paris in 1907, the society gave way the secret nationalist society of Abhinav Bharat Mandal, founded by V.D. Savarkar. The society was founded amongst efforts and movements that arose to reverse the flow of authority and power from Britain to India. along with substantial help from Bhikaji Cama. Think · Abhinav Bharat Mandal · V D Savarkar

QUESTION 74. National Council of Education and System was founded in 1906. Which of the following statements regarding the council are correct? 1. It was founded by the British Government in Delhi 2. The objective of the council was to promote education in science and Technology. 3. Institutions functioning under the Council were considered to be hotbeds of swadeshi activities and the government banned nationalistic activities such as the singing of patriotic songs Select the code from following: a) 1 and 2 b) 2 and 3 c) 1 and 3 d) All of the above

Correct Answer: B Explanation Solution (b) The National Council of Education (or NCE ) was an organisation founded by Indian nationalists in Bengal in 1906 to promote science and technology as part of a swadeshi industri alisation movement. It established the Bengal National College and Bengal Technical Institute which would later merge to form Jadavpur University. Institutions functioning under the Council were considered to be hotbeds of swadeshi activities and the government banned nationalistic activities such as the singing of patriotic songs. Think · Indian Universities Act 1904

QUESTION 54. Ghadar movement started as a revolutionary movement against the British Rule. The agenda of Ghadar Party was built around a weekly newspaper 'The Ghadar'. Which of the following statements regarding 'Ghadar' are correct? 1. It was published in US and Britain 2. It was published in Urdu and Punjabi 3. It carried a caption on the masthead - 'Angrezi Raj ka Dushman' Select the code from following: a) 1 and 2 b) 2 and 3 c) 1 and 3 d) All of the above

Correct Answer: B Explanation Solution (b) The party was built around the weekly paper The Ghadar , which carried the caption on the masthead: Angrezi Raj Ka Dushman (an enemy of the British rule). "Wanted brave soldiers", the Ghadar declared, "to stir up rebellion in India. Pay-death; Price-martyrdom; Pension-liberty; Field of battle-India". The ideology of the party was strongly secular. In the words of Sohan Singh Bhakna, who later became a major peasant leader of the Punjab: "We were not Sikhs or Punjabis. Our religion was patriotism". The first issue of The Ghadar , was published from San Francisco on November 1, 1913. Following the voyage of the Komagata Maru in 1914, a direct challenge to Canadian anti-Indian immigration laws, several thousand Indians resident in the United States sold their business and homes ready to drive the British from India. However, Hardayal had fled to Europe concerned that the US authorities would hand him over to the British. Sohan Singh Bhakna was already in British hands, and the leadership fell to Ram Chandra. Following the entry of Canada into World War I, the organisation was centred in the USA and received substantial funding from the German government. They had a very militant tone, as illustrated by this quote from Harnam Singh: "No pundits or mullahs do we need" The party rose to prominence in the second decade of the 20th century, and grew in strength owing to Indian discontent over World War I and the lack of political reforms. Ghadar activists undertook what the British described as political terrorism. Ghadar activists were responsible for bombs planted on government property. In 1917 some of their leaders were arrested and put on trial in the Hindu German Conspiracy Trial in which their paper was quoted. The Ghadar party commanded a loyal following the province of Punjab, but many of its most prominent activists were forced into exile to Canada and the United States. It ceased to play an active role in Indian politics after 1919. The party had active members in other countries such as Mexico, Japan, China, Singapore, Thailand, Philippines, Malaya, Indo-China and Eastern and Southern Africa. Think · Indian Nationalist movement in foreign countries

QUESTION 23. Vaikom Satyagraha was related to a) High rents extracted from the peasants b) Temple entry for depressed classes c) Implement recommendations of flood commission d) Oppression of tenants by Landlords.

Correct Answer: B Explanation Solution (b) Vaikom Satyagraha was a movement in Travancore (modern-day Kerala) for temple entry of the depressed classes. It took place near the Shiva Temple at Vaikom, Kottayam district, Kerala during 1924-25. Vaikom was at that time a part of the princely state of Travancore. Do you know? · Tebhaga movement was launched in Bengal to implement recommendations of Flood Commission to give two-third (tebhaga) share to the bargardars (share-croppers) instead of one-half share. THINK! · Eka movement · Mopillah revolt

QUESTION 42. A new mobile application 'LaQshya' has been launched by Government of India nationwide. What is the purpose of this application? a) It is an app for school students for career counseling b) It is app for health workers to carry safe normal or complicated child deliveries c) It is app to check implementation of MGNREGA in rural areas d) It is an app to set vocational training Quality check in ITIs across India.

Correct Answer: B Explanation Solution (b) • A Safe Delivery Mobile Application for health workers who manage normal and complicated deliveries in the peripheral areas, and also released the Operational Guidelines for Obstetric High Dependency Units (HDUs) and Intensive Care Units (ICUs). • It is expected to improve the quality of care that is being provided to the pregnant mother in the Labour Room and Maternity Operation Theatres, thereby preventing the undesirable adverse outcomes associated with childbirth. • This initiative will be implemented in Government Medical Colleges (MCs) besides District Hospitals (DHs), and high delivery load Sub- District Hospitals (SDHs) and Community Health Centres (CHCs). • The initiative plans to conduct quality certification of labour rooms and also incentivize facilities achieving the targets outlined. • The goal of this initiative is to reduce preventable maternal and new-born mortality, morbidity and stillbirths associated with the care around delivery in Labour room and Maternity OT and ensure respectful maternity care. • Concerned Ministry - Ministry of Health and Family Welfare Safe Delivery Application • The Safe Delivery Application is a mHealth tool that can be used for health workers who manage normal and complicated deliveries in the peripheral areas. • The application has Clinical Instruction films on key obstetric procedures which can help the health workers translate their learnt skills into practice. • It can play a pivotal role in training, post training reinforcement, mentoring and demonstration. • This application has been tailored as per the Indian context. • It has also been field tested in few districts and has been found to be useful for health workers who provide maternity care http://www.newindianexpress.com/pti-news/2017/dec/11/govt-launches-initiative-to-implement- labour-room-guidelines-1724646.html

QUESTION 20. Gandhi was not in support of an immediate struggle in 1939 after British declared India's support to the war without consulting Indians. Which of the following were the reasons for his stand? 1. Gandhi felt Allied cause was just. 2. Communal sensitivity and lack of Hindu-Muslim unity at the time. 3. Masses were not ready for struggle. 4. Congress organization was not fit enough to carry a mass struggle. Select the correct code a) 1 and 2 b) 1,2 and 3 c) 1 and 3 d) 1,2,3 and 4

Correct Answer: D Explanation Solution (d) Gandhi and his supporters were not in favour of an immediate struggle because they felt that the · Allied cause was just; · Communal, sensitivity and lack of Hindu-Muslim unity could result in communal riots; · Congress organisation was in shambles and the atmosphere was not conducive for a mass struggle; · Masses were not ready for a struggle. They instead advocated toning up the Congress organisation, carrying on political work among the masses, and negotiating till all possibilities of a negotiated settlement were exhausted. Only then would the struggle be begun. Do you know? · Subhash Chandra Bose and Leftists wanted to take advantage of Britain's difficulties and start a mass movement, but Gandhi was for unconditional support to Britain's War efforts. THINK! · Linlithgow's Statement (October 1939)

QUESTION 83. Consider the following statements regarding 'Tolstoy Farm': 1. Tolstoy farm was setup by Gandhi ji in Gujrat on the Bank of Sabarmati river. 2. The farm was made to house Satyagrahis 3. Schooling was provided to children and youths by Gandhi ji himself. Which of the above statements are correct? a) 1 and 2 b) 2 and 3 c) 1 and 3 d) All of the above

Correct Answer: B Explanation Solution (b) Tolstoy Farm Tolstoy Farm was a community started by Gandhi in Transvaal, South Africa, in 1910. It became the headquarters of the campaign of satyagraha (non-violence) which he lead at that time. This campaign was a reaction to the discrimination against Indians in Transvaal, and at one stage, 2,500 people of Indian origin were in jail for non-violent resistance to the racist laws. The farm, 22 miles from Johannesburg, had been bought by Herman Kallenbach, one of Gandhi's supporters and placed at the disposal of the Satyagrahis for as long as the campaign lasted. Gandhi recalled that between 70 and 80 people lived in this "co-operative commonwealth", but sometimes there were more. They were mostly young men and children, with a few women and older men. They were members of various religions and various origins, not just Indians. Various languages, Gujarati, Tamil, Hindi and English, were spoken by the residents. Adults and children worked in farming and gardening, including pruning fruit trees, growing, harvesting, and clearing woodland. The emphasis was upon simple communal living combined with a clearly structured routine of work, school, bathing and a general meeting at the end of the day. Meals were vegetarian. Think · My Experiments with Truth · Satyagraha in South Africa

QUESTION 99. Tuitial Hydro Power Project is located in a) Assam b) Manipur c) Mizoram d) Arunachal Pradesh

Correct Answer: C Explanation Solution (c) 60 MW Tuitial Hydroelectric Power Project in Mizoram The Tuirial HEPP has been constructed as a Central Sector Project and implemented by North Eastern Electric Power Corporation (NEEPCO), under the administrative control of the Ministry of Power, Government of India. http://pib.nic.in/newsite/PrintRelease.aspx?relid=174336

QUESTION 16. Which of the following were the features of Government of India Act of 1935? 1. It proposed an All-India Federation that started functioning after 1937 elections. 2. Federal Assembly was indirectly elected whereas Council of states was directly elected. 3. For legislation purposes, three lists were introduced, namely federal, provincial and concurrent. Select the correct code a) 1 and 2 b) 1 and 3 c) 2 and 3 d) 1,2 and 3

Correct Answer: C Explanation Solution (c) An All India Federation: It was to comprise all British Indian provinces, all chief commissioner's provinces and Indian states. The federation's formation was conditional on the fulfilment of two conditions: (i) States with allotment of 52 seats in the proposed Council of States should agree to join the federation; (ii) Aggregate population of states in the above category should be 50 per cent of the total population of all Indian states. Since these conditions were not fulfilled, the proposed federation never came up . The Central Government carried on upto 1946 as per the provisions of Government of India Act, 1919. The bicameral legislature was to have an upper house (Council of States) and a lower house (Federal Assembly). The Council of States was to be a 260-member house, partly directly elected from British Indian provinces and partly (40 per cent) nominated by the princes. The Federal Assembly was to be a 375-member house, partly indirectly elected from British Indian provinces and partly (one-third) nominated by the princes. Oddly enough, election to the Council of States was direct and that to the Federal Assembly, indirect. Council of States was to be a permanent body with one-third members retiring every third year. The duration of the assembly was to be 5 years. The three lists for legislation purposes were to be federal, provincial and concurrent. Do you know? · The Act enfranchised 14 per cent of British Indian population. · The Act provided a rigid Constitution with no possibility of internal growth. Right of amendment was reserved with the British Parliament. THINK! · Salient features of Government of India Act 1935 · Features of Indian constitution borrowed from 1935 Act

QUESTION 62. Which of the following statements is/are correct regarding Swadesh Bandhab Samiti? 1. The samiti was founded by Ashwini Kumar Dutta. 2. The objective of the Samiti was to promote Swadeshi and boycott foreign goods. Select the code from following: a) 1 only b) 2 only c) Both 1 and 2 d) Neither 1 nor 2

Correct Answer: C Explanation Solution (c) Ashwini Kumar Dutta was a Bengali educationist , philanthropist, social reformer and patriot. He founded the Swadesh Bandhab Samiti to promote the consumption of indigenous products and boycott foreign goods. When the moderates and the extremists parted ways in the Surat session, he attempted a reconciliation between the two groups.

QUESTION 21. Which of the following were proposed by Cabinet Mission? 1. Grouping of provincial assemblies into four sections. 2. Full autonomy and residual powers to provinces. 3. Constituent Assembly to be elected by provincial assemblies by proportional representation. Select the correct code a) 1 and 2 b) 1 and 3 c) 2 and 3 d) 1,2 and 3

Correct Answer: C Explanation Solution (c) Cabinet Mission Plan—Main Points · Rejection of the demand for a full-fledged Pakistan · Grouping of existing provincial assemblies into three Sections § Section-A: Madras, Bombay, Central Provinces, United Provinces, Bihar and Orissa (Hindu- majority provinces). § Section-B: Punjab, North-West Frontier Province and Sindh (Muslimmajority provinces). § Section-C: Bengal and Assam (Muslim-majority provinces). Three-tier executive and legislature at provincial, section and union levels. · A constituent assembly to be elected by provincial assemblies by proportional representation (voting in three groups—General, Muslims, Sikhs). · A common centre would control defence, communication and external affairs. Communal questions in central legislature were to be decided by a simple majority of both communities present and voting. · Provinces were to have full autonomy and residual powers . Princely states were no longer to be under paramountcy of British Government They would be free to enter into an arrangement with successor governments or the British Government. · After the first general elections, a province was to be free to come out of a group and after 10 years, a province was to be free to call for a reconsideration of the group or the Union constitution. · Meanwhile, an interim government to be formed from the constituent assembly. Do you know? · Despite different interpretations and objections, both Congress and Muslim league accepted the plan, which resulted in formation of Constituent Assembly. THINK! · Reasons for introducing Cabinet Mission Plan · Objections to the plan

QUESTION 26. Which of the following were the terms of Gandhi-Irwin Pact? 1. Release of all political prisoners 2. Participation of congress in Second Round table conference 3. Right to make salt for personal consumption Select the correct code a) 1 and 2 b) 1 and 3 c) 2 and 3 d) 1,2 and 3

Correct Answer: C Explanation Solution (c) GANDHI-IRWIN PACT On January 25, 1931 Gandhi and all other members of the CWC were released unconditionally. The CWC authorised Gandhi to initiate discussions with the viceroy. As a result of these discussions,apact was signed between the viceroy, representing the British Indian Government, and Gandhi, representing the Indian people, in Delhi on February 14, 1931. This Delhi Pact, also known as the Gandhi-Irwin Pact, placed the Congress on an equal footing with the Government. Irwin on behalf of the Government agreed on 1. Immediate release of all political prisoners not convicted of violence ; 2. Remission of all fines not yet collected; 3. Return of all lands not yet sold to third parties; 4. Lenient treatment to those government servants who had resigned; 5. Right to make salt in coastal villages for personal consumption (not for sale ); 6. Right to peaceful and non-aggressive picketing; and 7. Withdrawal of emergency ordinances. The viceroy, however, turned down two of Gandhi's demands— (i) Public inquiry into police excesses, (ii) Commutation of Bhagat Singh and his comrades' death sentence to life sentence. Gandhi on behalf of the Congress agreed— (i) To suspend the civil disobedience movement, and (ii) To participate in the next RTC on the constitutional question around the three Finch-pins of federation, Indian responsibility, and reservations-and safeguards that may be necessary in India's interests (covering such areas as defence, external affairs, position of minorities, financial credit of India and discharge of other obligations). Do you know? · When Gandhi met Irwin and held negotiations, it was for the first time that the two were meeting as 'equals'. THINK! · Civil disobedience movement · Round Table Conferences

QUESTION 27. Hartog committee appointed in 1929 was related to which of the following sectors? a) Freedom of Press b) Police Reforms c) Education d) Revenue sharing with provinces

Correct Answer: C Explanation Solution (c) HARTOG COMMITTEE (1929) An increase in number of schools and colleges had led to deteriorationof education standards. A Hartog Committee was set up to report on development of education. Its main recommendations were as follows. 1. Emphasis should be given to primary education but there need be no hasty expansion or compulsion in education. 2. Only deserving students should go in for high school and intermediate stage, while average students should be diverted to vocational courses after. VIII standard. 3. For improvements in standards of university education, admissions should be restricted. Do you know? · Indians also attempted to develop an indigeneous scheme of education called Wardha scheme of education based on Gandhi's ideas under chairmanship of Dr. Zakir Hussain. THINK! · Educational reforms under British India

QUESTION 46. Recently INS Kalvari has been inducted in the Navy by Prime Minister of India. Which of the following statements about INS Kalvari are correct? 1. It is a diesel - electric attack submarine. 2. The project has been undertaken in collaboration with Israel. 3. It can launch torpedoes while on the surface or being submerged. Select the code from following: a) 1 and 2 b) 2 and 3 c) 1 and 3 d) All of the above

Correct Answer: C Explanation Solution (c) INS Kalvari • It is a diesel-electric attack submarine that has been built for the Indian Navy by the Mazagon Dock Shipbuilders Limited. • It is the first of six such submarines that will be inducted into the Indian Navy, and represents a significant success for the "Make in India" initiative. • The project has been undertaken with French collaboration. • The Kalvari will be the most modern non-nuclear submarine in the Navy. • The submarine is armed with heavy weight torpedoes and Exocet anti-ship missiles. The submarine can launch torpedoes both while submerged or on the surface. • The heart of the Kalvari's weapons system is SUBTICS or Submarine Tactical Integrated Combat System suite which processes information from the sonars onboard the submarine in detecting targets which can then be engaged with torpedoes or missiles • The submarine has an attack and search periscope equipped with Infrared/Low Light Level cameras and Laser Range finders to spot targets on the surface of the sea. Think • Scorpene class submarines • Project-75 http://pib.nic.in/newsite/PrintRelease.aspx?relid=174292

QUESTION 73. Congress session in 1906 was held in Calcutta and presided by Dada Bhai Naoroji. Which of the following resolutions were taken by the congress in this session? 1. Resolution on Partition of Bengal 2. Resolution on Independence of India 3. Resolution on Swadeshi 4. Resolution on Boycott Select the code from following: a) 1, 2 and 3 b) 2,3 and 4 c) 1, 3 and 4 d) All of the above

Correct Answer: C Explanation Solution (c) In 1906, the session at Calcutta was presided by Dada Bhai Naoroji. The moderates chose Dada Bhai Naoroji to preside the Congress. Dada Bhai Naoroji, the Grand Oldman of India was respected by the moderates and extremists alike. But, in this session, the congress was compelled by the extremists to adopt following resolutions which were accepted by the moderates with half heart. These were as follows: 1. Resolution on Partition of Bengal 2. Resolution of Self Government (Swaraj) 3. Resolution on Swadeshi 4. Resolution on Boycott. Thus in Calcutta session of 1906, under the leadership of Dadabhai Naoroji, Congress adopted Swaraj as the Goal of Indian people. But here a bit of politics was played by the Moderates. They, by no means wanted to be tagged as radical. The toned down the resolution in a compromised state and made it " self government means that obtaining the self governing British Colonies ". Thus the whole meaning of Swaraj of the extremists was changed. Think · When did INC took complete independence as a resolution? · Difference between 'Swaraj' and complete independence

QUESTION 25. 'A plan was proposed to declare the provinces as independent states and then giving them option to choose whether to join India or Pakistan or remaining separate'. What was this plan called? a) Desai-Liaqat Pact b) Wavell's Breakdown Plan c) Plan Balkan d) CR Formula

Correct Answer: C Explanation Solution (c) In May 1947, Mountbatten came up with a plan under which he proposed that the provinces be declared independent successor states and then be allowed to choose whether to join the constituent assembly or not. This plan was called the 'Dickie Bird Plan'. Jawaharlal Nehru, when apprised of the plan, vehemently opposed it saying it would lead to balkanisation of the country. Hence, this plan was also called Plan Balkan. Do you know? · Wavell's Breakdown Plan was presented before the Cabinet Mission and was different than Wavell Plan, which was decided in Shimla Conference. THINK! · CR Formula · Desai Liaqat Pact

QUESTION 89. Despite strict laws, the illegal trade of 'Pangolin' is threatening the very existence of the species. What is the status of India Pangolin under IUCN? a) Near Threatened b) Least Concern c) Endangered d) Extinct in the Wild

Correct Answer: C Explanation Solution (c) Indian Pangolin is listed as Endangered because it is subject to hunting and increasing levels of poaching, principally for its meat and scales, both for local use and for illicit international trade in scales, which has also occurred historically. Available evidence suggests this trade is destined for East Asia where scales are used in traditional medicines. Reference- https://economictimes.indiatimes.com/news/environment/flora-fauna/pangolin- traffickers-opening-up-new-routes-study/articleshow/62082327.cms http://www.iucnredlist.org/details/12761/0

QUESTION 45. Which of the following statements are correct regarding Indian Technical and Economic Cooperation Program (ITEC)? 1. It is a bilateral assistance program run by Government of India. 2. It is a response oriented program that focuses on addressing the needs of Developed nations. 3. ITEC covers 158 countries across Asia, Africa, Latin America, Central and Eastern Europe, and several Pacific and Caribbean nations. Select the code from following: a) 1 and 2 b) 2 and 3 c) 1 and 3 d) All of the above

Correct Answer: C Explanation Solution (c) Indian Technical and Economic Cooperation Program (ITEC) • It is a bilateral assistance programme run by the Government of India. • It is a demand-driven, response-oriented programme that focuses on addressing the needs of developing countries through innovative technological cooperation between India and the partnering nation. • Along with its corollary the Special Commonwealth Assistance for Africa Programme, ITEC covers 158 countries across Asia, Africa, Latin America, Central and Eastern Europe, and several Pacific and Caribbean nations. Components • Training (civilian and defence) in India of nominees from ITEC partner countries; • Projects and project related activities such as feasibility studies and consultancy services; • Deputation of Indian experts abroad; • Study Tours; • Gifts/Donations of equipment at the request of ITEC partner countries; and • Aid for Disaster Relief. DPA-II Division of Development Partnership Administration (DPA) in the Ministry of External Affairs is the nodal division for handling all capacity building programmes. Think • Special Commonwealth African Assistance Programme (SCAAP) • Colombo Plan • South-South Cooperation

QUESTION 86. Rajiv Gandhi National Institute of Youth Development (RGNIYD) is an Institution of National Importance by the Act of Parliament, located at a) New Delhi b) Ahmedabad c) Gwalior d) Tamilnadu

Correct Answer: D Explanation Solution (d) The Rajiv Gandhi National Institute of Youth Development (RGNIYD), Sriperumbudur, Tamil Nadu, is an Institution of National Importance by the Act of Parliament. Brought out the India Youth Development Index and Report 2017 which is a pioneering attempt made by the Institute in 2010 and followed it up with the India Youth Development Index in 2017. Objectives of RGNIYD · To evolve as an Institute of advanced study in the field of youth development. · To undertake action and applied research · To empower youth to participate in inclusive development and nation building · To provide higher education in the field of youth development · To carry out policy research, evaluation and impact analysis of youth programmes

QUESTION 40. 'The State of the World's Children' is an annual report published by a) Ministry of Women and Child development b) WHO c) UNICEF d) World Economic Forum

Correct Answer: C Explanation Solution (c) It is an annual report published by the United Nations Children's Emergency Fund (UNICEF). It is the flagship publication of the organization. The State of the World's Children 2017: Children in a Digital World • It presents UNICEF's first comprehensive look at the different ways digital technology is affecting children's lives and life chances, identifying dangers as well as opportunities. • It argues that governments and the private sector have not kept up with the pace of change, exposing children to new risks and harms and leaving millions of the most disadvantaged children behind. • Around one third of the world's youth - 346 million - are not online, exacerbating inequities and reducing children's ability to participate in an increasingly digital economy. • One in three Internet users in the world is a child. • The report also examines how the internet increases children's vulnerability to risks and harms, including misuse of their private information, access to harmful content, and cyberbullying. • Digital networks like the Dark Web and cryptocurrencies are enabling the worst forms of exploitation and abuse, including trafficking and 'made to order' online child sexual abuse. • In India only 29% of all Internet users are female • Girls in rural areas often face restrictions on their use of ICTs solely because of their gender. • Worldwide, 71% are online compared with 48% of the total population. African youth are the least connected, with around 3 out of 5 youth offline, compared to just 1 in 25 in Europe http://indianexpress.com/article/technology/tech-news-technology/one-in-three-internet-users- in-the-world-a-child-says-unicef-4978791/

QUESTION 90. Recent announcement by US President giving recognition of 'Jerusalem' as the capital of Israel sparked controversy all over the World. Consider the following 1. Jerusalem 2. Dead Sea 3. Mediterranean Sea Select the correct order of location of above entities arranging them from East to West direction: a) Mediterranean Sea- Jerusalem- Dead Sea b) Jerusalem- Dead Sea- Mediterranean Sea c) Dead Sea- Jerusalem- Mediterranean Sea d) Jerusalem-Mediterranean Sea- Dead Sea

Correct Answer: C Explanation Solution (c) Jerusalem, a Middle Eastern city west of the Dead Sea, has been a place of pilgrimage and worship for Jews, Christians and Muslims since the biblical era.

QUESTION 34. Which of the following personalities started a newspaper called 'Swaraj'? a) B.G. Tilak b) Sardar Vallabhai Patel c) Subhash Chandra Bose d) Motilal Nehru

Correct Answer: C Explanation Solution (c) Subhash Chandra Bose started the newspaper called 'Swaraj'. Do you know? · He was selected for the Indian Civil Services (ICS) but refused to take up service since he did not want to serve the British government. THINK! · Contributions of Subhash Chandra Bose in freedom struggle · Newspapers by important personalities of freedom struggle

QUESTION 51. 'The Arctic home in the Vedas' is a history book on the origin of the Aryans. Who of the following personalities authored this book? a) Annie Besant b) Jawaharlal Nehru c) Bal Gangadhar Tilak d) Lala Lajpat Rai

Correct Answer: C Explanation Solution (c) The Arctic Home in the Vedas is a history book on the origin of Aryanic People by Bal Gangadhar Tilak, a mathematician turned astronomer, historian, journalist, philosopher and political leader of India. It propounded the theory that the North Pole was the original home of Aryans during the pre-glacial period which they had to leave due to the ice deluge around 8000 B.C. and had to migrate to the Northern parts of Europe and Asia in search of lands for new settlements. In support to his theory, Tilak presented certain Vedic hymns, Avestic passages, Vedic chronology and Vedic calendars with interpretations of the contents in detail. The book was written at the end of 1898, but was first published in March 1903 in Pune. Think · Origin of Aryans

QUESTION 79. The Indian Sociologist was an Indian nationalist journal in the early 20th century. Its subtitle was An Organ of Freedom, and Political, Social, and Religious Reform . Who of the following was editor of this journal? a) Bikajicama b) Dorezio c) Shyamji Krishnavarma d) M N Roy

Correct Answer: C Explanation Solution (c) The Indian Sociologist was an Indian nationalist journal in the early 20th century. Its subtitle was 'An Organ of Freedom, and Political, Social, and Religious Reform'. The journal was edited by Shyamji Krishnavarma from 1905 to 1914, then between 1920 and 1922. It was originally produced in London until May 1907 when Krishnavarma moved to Paris. The journal was edited in Paris from June 1907, but the change of address was only announced in the September 1907 issue. Publication continued in Paris until 1914, when Krishnavarma moved to Geneva on account of the First World War. While in Geneva he abandoned the publication under pressure from the Swiss authorities. He recommenced publication in December 1920 and continued until September 1922. Think · Impact of rise of communism in Russia on Indian National Movement

QUESTION 6. Consider the following statements 1. The Khilafat movement was a pan-Islamic, political protest campaign launched by Muslims of India to influence the British government not to abolish the Ottoman Caliphate. 2. Khilafat Movement was started against Turkish ruler Mustafa Kemal Ataturk. Which of the statements given above are correct? a) 1 only b) 2 only c) Both d) None

Correct Answer: C Explanation Solution (c) The Khilafat movement (1919-22) was a pan-Islamic, political protest campaign launched by Muslims of India to influence the British government not to abolish the Ottoman Caliphate. The movement collapsed by late 1922 when Turkey gained a more favourable diplomatic position and moved toward secularism. By 1924 Turkey simply abolished the roles of the Sultan and Caliph. · The Ali brothers were arrested during the course of the war only to be released from jail after the war was over. This movement reached a crescendo in Bengal, North-West Frontier Province and the Punjab. · October 17, 1919 was observed as Khilafat Day. The Hindus also joined hands with the Muslims and a strike was called for. · On November 23 1919, the All India Khilafat Conference was organized at New Delhi and later a Khilafat Manifesto was published which called upon the British to protect the Caliphate. Do you know? · Khilafat and Non-Cooperation movement, while bringing Muslims into national movement also led to communalization of national politics. After 1922 a series of differences between the Khilafat and Non-Cooperation leaderships intersected with growing popular conflict between Hindu and Muslim communities. Some section of Muslims started to see the futility of Swaraj and fresh interest was awakened in the Muslim League which had been stagnant since 1918. · The Ahmedabad session of Congress in 1921 was presided over by C.R. Das while still in jail. THINK! · Reasons for launch of movement · Chauri chaura incident and reasons for abrupt withdrawal of movement

QUESTION 55. Recently Justin Trudeau, the Prime Minister of Canada, apologized for Komagata Maru Incident. What was Komagata Maru? a) German political party based in Canada b) Japanese tribe which migrated to Canada during World War I c) A Ship on voyage to Canada d) A Japanese island attacked by Canadian forces

Correct Answer: C Explanation Solution (c) The Komagata Maru incident involved the Japanese steamship Komagata Maru on which a group of citizens of the British Raj attempted to emigrate to Canada in 1914 but were denied entry. Komagata Maru sailed from British Hong Kong, via Shanghai, China, and Yokohama, Japan, to Vancouver, British Columbia, Canada, in 1914, carrying 376 passengers from Punjab, British India. Of them, 24 were admitted to Canada, but the other 352 passengers were not allowed to disembark in Canada, and the ship was forced to return to India. The passengers comprised 340 Sikhs, 24 Muslims, and 12 Hindus, all British subjects. This was one of several incidents in the early 20th century in which exclusion laws in Canada and the United States were used to exclude immigrants of Asian origin. Think · Why Indians were denied entry in Canada?

QUESTION 91. Recently MI-8 has been phased out of Indian Army. What is MI-8? a) Fighter plane b) Tank c) Missile d) Helicopter

Correct Answer: D Explanation Solution (d) The Soviet-era MI-8, the backbone of the Indian Air Force helicopter operations, was phased out, bringing an end to its glorious service career spanning 45 years. http://www.thehindu.com/news/national/iafs-mi-8-chopper-flies-into-history/article21827710.ece

QUESTION 43. Which of the following statements is/are correct regarding India's National Informatics Centers - Computer Emergency Response Team (NIC - CERT)? 1. It is an initiative of Ministry of Electronics and Information Technology. 2. NIC-CERT has been setup with the objective of creating a comprehensive framework that integrates world class security components and inbuilt threat intelligence for detection, prevention and incident response. Select the code from below: a) 1 only b) 2 only c) Both 1 and 2 d) Neither 1 nor 2

Correct Answer: C Explanation Solution (c) • A setup of National Informatics Centre, Ministry of Electronics and Information Technology. • It is MeitY's initiative under Digital India aimed at enhancing the security posture of NIC and the Government which will in turn lead to enhanced trust of the citizens as the services offered to them would be configured in a framework that is secure by design. • NIC-CERT will operate in close coordination and collaboration with sectoral CERT's and more so with Cert-in. • NIC-CERT has been setup with the objective of creating a comprehensive framework that integrates world class security components and inbuilt threat intelligence for detection, prevention and incident response. Do You Know? • The National Informatics Centre (NIC) helps government agencies build their digital presence while CERT-India is the official agency of the government to track, prevent and tackle cyber threats. Think • Indian Computer Emergency Response Team (CERT-In) http://www.business-standard.com/article/government-press-release/sh-ravi-shankar-prasad- launches-first-nic-cert-cert-to-prevent-117121100860_1.html

QUESTION 71. Al - Hilal was an Urdu newspaper published by Maulana Abul Kalam Azad. Which of the following statements regarding Al Hilal is/are correct? 1. The newspaper urged Muslims to join Nationalist Movement and criticized British Government 2. The newspaper was shut down under the Press Act of 1914. Select the code from following: a) 1 only b) 2 only c) Both 1 and 2 d) Neither 1 nor 2

Correct Answer: C Explanation Solution (c) Al - Hilal The Al-Hilal was a weekly Urdu language newspaper established by the Indian leader Maulana Abul Kalam Azad and used as a medium for criticism of the British Raj in India. The first issue came out on 13 July 1912. The newspaper also espoused the cause of the Indian independence movement and exhorted Indian Muslims to join the movement. The newspaper was shut down under the Press Act of 1914. "Al-Hilal (The Crescent), published in Calcutta, ushered in a new chapter in Urdu journalism and immediately appealed to Muslims in the city". The Al- Hilal covered a range of issues related with theology, politics, wars and scientific advancement besides its critical coverage of the Raj in India and it went on to become a very popular newspaper, reaching a peak circulation of over 25,000, a new record for Urdu journalism in those days. The paper played a catalytic role in shaping Muslim opinion against the Raj, a fact acknowledged by many stalwarts of India's freedom struggle. Think · Make a list of the journals and newspapers published by the prominent Indian leaders.

QUESTION 95. Consider the following statements 1. Pattachitra is an art form belonging to Raghurajpur village of Odisha. 2. Pattachitra has been awarded Geographical Indication tag 3. These paintings are made on a canvas, which is prepared by mashing an old cotton cloth and palm leaves. Which of the given statement/s is/are correct? a) 1 only b) 2 and 3 c) 1 and 3 d) 1, 2 and 3

Correct Answer: D Explanation Solution (d) It is the traditional cloth-based scroll painting of Odisha and was given the GI tag in 2008. The paintings are generally based on Hindu mythology and resemble the old murals of the state. These paintings are made on a canvas, which is prepared by mashing an old cotton cloth and palm leaves. When the canvas dries up , it is hardened using a paste of tamarind, turmeric, chalk and granite powder. The colours used in these paintings are made from from coal, conch shells, turmeric, chalk powder, leaves of selected plants and soft stones. https://timesofindia.indiatimes.com/city/allahabad/old-yet-revered-pattachitra-art-grabs- eyeballs-at-shilp-mela/articleshow/62016174.cms

QUESTION 85. Ahrar Movement was founded in 1910 in Punjab. Which of the statements are correct about this movement? 1. It was founded by Hakim Ajmal khan 2. The movement was against the loyalist politics of the Aligarh movement. 3. Ahrar movement advocated the active involvement of Muslims in Indian nationalist movement. Select the code from following: a) 1 and 2 b) 2 and 3 c) 1 and 3 d) All of the above

Correct Answer: D Explanation Solution (d) Ahrar Movement Ahrar Movement was a movement founded in 1910 under the leadership of Maulana Muhammed Ali Hakim Ajmal Khan. This movement was in opposition to the loyalist politics of the Aligarh movement, its members advocated active participation in the nationalist movement. Ahrar movement was moved by the modern ideas of self-government, its member advocated active participation in the nationalist movement. Hakim Ajmal Khan, Iqbal, CR Das, Sir Mian Muhammad Shafi and leaders of Ahrar were against the Lucknow Pact. The issue of separate electorates got prominence in the Muslim majority provinces by the late 1920s due to the unwise weightage formula of the Lucknow Pact. Think Why were the leaders of Ahrar Movement against Muslim League?

QUESTION 94. Consider the following statements regarding 'Chitrakathi' 1. Chitrakathi is a rare folk art practised in Pinguli, a small village near Kudal in the Sindhudurg district of Maharashtra. 2. Performed by Takars, a nomadic clan, Chitrakathi dates back to 17th century. 3. Painting is done using paper, brush and hand-made colours and in the sequence, based on the story of Mahabhartha or Ramayana. Which of the given statement/s is/are correct? a) 1 only b) 2 and 3 c) 1 and 3 d) 1, 2 and 3

Correct Answer: D Explanation Solution (d) All the statements are correct Chitrakathi is a rare folk art practised in Pinguli, a small village near Kudal in the Sindhudurg district of Maharashtra. Performed by Takars, a nomadic clan, Chitrakathi dates back to 17th century. Pictures were used to narrate the story in Chitrakathi. "Paintings were made in sequence (based on the story of Mahabharatha) using paper, brush and hand- made colours. Traditionally, chitrakathi art was created on festive occasions in village temples, accompanied by oral renditions of the Ramayana or Mahabharata. The story would be rendered in song backed by instruments along with the relevant pictures. But unfortunately, the art form is on the verge of extinction due to lack of patronage. Do You Know? · Bharatha-k-Koothu is widely performed in the Draupathi Amman temples in the northern part of Tamil Nadu.In Bharatha-k-Koothu, the artists dressed in the traditional attire narrate the tale of Mahabharatha through songs and dialogues. http://www.business-standard.com/article/government-press-release/leela-an-exhibition-on-the- ramayana-tradition-being-held-at-the-117121700578_1.html

QUESTION 87. Consider the following statements 1. The National Energy Conservation Day is being celebrated every year on December 14 since 1991. 2. The Bureau of Energy Efficiency (BEE), under Ministry of Power spearheads the celebrations every year. 3. The objective to celebrate the National Energy Conservation Day is to drive mass awareness about the importance of energy efficiency and conservation. Which of the given statement/s are correct? a) 1 and 3 b) 2 and 3 c) 1 and 2 d) 1, 2 and 3

Correct Answer: D Explanation Solution (d) All the statements are correct. National energy conservation day is celebrated every year by the people all over the India on 1 4th of December . The Energy Conservation Act in India was executed by the Bureau of Energy Efficiency (BEE) in the year 2001. The Bureau of Energy Efficiency is a constitutional body which comes under Government of India and helps in the development of policies and strategies in order to reduce the energy use. National energy conservation day in India is celebrated to aware people about the importance of energy as well as saving or conserving the more energy by using less energy. The exact means of energy conservation is using less energy by avoiding the unnecessary uses of energy. Using energy efficiently is very necessary to save it for the future usage. Energy conservation should be rooted in the behavior of every human being to get more effect towards the plan of energy conservation. https://economictimes.indiatimes.com/industry/energy/power/india-to-auction-5gw-offshore- wind-projects-in-2018/articleshow/62068158.cms

QUESTION 29. Which of the following were the fundamental causes of famine of 1943? 1. Food was diverted to feed Army. 2. Rice imports from Burma and South -East Asia were stopped. 3. Deliberate profiteering and mismanagement of food resources. Select the correct code a) 1 and 2 b) 1 and 3 c) 2 and 3 d) 1,2 and 3

Correct Answer: D Explanation Solution (d) Bengal famine of 1943: The worst affected areas were southwest Bengal comprising the Tamluk- Contai-Diamond Harbour region, Dacca, Faridpur, Tippera and Noakhali. Around 1.5 to 3 million people perished in this basically man-made famine, the epidemics (malaria, cholera, small pox), malnutrition and starvation. The fundamental causes of the famine were as follows. 1. The need to feed a vast Army diverted foodstuffs. 2. Rice imports from Burma and South-East Asia had been stopped due to fear of Japanese invasion. 3. Gross mismanagement and deliberate profiteering aggravated the famine; rationing methods were belated and were confined to big cities. Do you know? · Despite having a detailed Famine Code t hat would have triggered a sizable increase in aid, the provincial government never formally declared a state of famine. THINK! · Major famines during British rule · Famine commission

QUESTION 49. In which of the following conditions can 'Cochlear implant' may help children? 1. Have severe to profound hearing loss in both ears. 2. Receive little or no benefit from hearing aid 3. No development, delay or mental retardation Select the code from following: a) 1 and 2 b) 2 and 3 c) 1 and 3 d) All of the above

Correct Answer: D Explanation Solution (d) Cochlear Implant • It is an electronic device having two parts (External and Internal). • Internal part is surgically implanted in the skull (Cochlear) by ENT surgeons. • Post-operative rehabilitation is undertaken by Audiologist and speech language pathologist and special educator of Hearing Impaired. • Parent have very important role in stimulating child to learn speech and language at home and maintain the external part (Processor). • Children with hearing loss as young as 12 months old and within 5 years of age can be eligible for a cochlear implant under ADIP scheme as per the criteria of ADIP Scheme of Govt. of India. The cochlear implant technology may help children who: • Have severe to profound hearing loss in both ears • Receive little or no benefit from hearing aids • No Development Delay or Mental Retardation ADIP Scheme • The main objective of the Assistance to Disabled persons for purchasing / fitting of aids / appliances (ADIP) scheme is to assist the needy disabled persons in procuring durable, sophisticated and scientifically manufactured, modern, standard aids and appliances that can promote their physical, social and psychological rehabilitation, by reducing the effects of disabilities and enhance their economic potential. • The aids and appliances supplied under the Scheme shall conform to BIS specifications to the extent possible. • The scheme is implemented through implementing agencies such as the NGOs, National Institutes under this Ministry and ALIMCO (a PSU). http://pib.nic.in/newsite/PrintRelease.aspx?relid=174318

QUESTION 68. Delhi Durbar was held in 1911 at the Red Fort. Which of the following statements are correct regarding Delhi Durbar? 1. Durbar was held to commemorate the coronation of King George V and Queen Mary as Emperor and Empress of India. 2. Capital of India was changed from Calcutta to Delhi 3. The partition of Bengal was cancelled. Select the code from following: a) 1 and 2 b) 2 and 3 c) 1 and 3 d) All of the above

Correct Answer: D Explanation Solution (d) Darbar was held to commemorate the coronation of King George V and Queen Mary as Emperor and Empress of India. The King and the Queen attired in the coronation robes with thousands of diamonds and precious gems appeared in the Jharokha of the red fort to give Darshan to 5 Lakh Indian people who gathered to greet the sovereign couple. · The Congress passed a resolution condemning the pomp and show of this Darbar at the cost of the poor Indians. · In this Darbar, the King declared that Capital of India will be transferred from Calcutta to Delhi. · In the same Darbar it was also declared the Partition of Bengal is cancelled. Think · How many Delhi Durbars were held in India? · What was the stand of Congress on Delhi Durbar in 1911?

QUESTION 30. Which of the following statements are correct about Dr. B.R. Ambedkar? 1. He was author of books like 'The Annihilation of Caste' and 'Pakistan or the Partition of India'. 2. He was founder of Independent Labour Party. 3. His death anniversary is observed as Mahaparinirvan Divas. Select the correct code a) 1 and 2 b) 1 and 3 c) 2 and 3 d) 1,2 and 3

Correct Answer: D Explanation Solution (d) Dr. B.R. Ambedkar was popularly known as Baba Saheb. He was the Chairman of the Drafting Committee of the Constituent Assembly and is called the 'Father of the Indian Constitution'. He established the Bahishkrit Hitakarini Sabha to promote education and socio-economic improvements among the Dalits. He also founded the Independent Labour Party (later transformed into the Scheduled Castes Federation) in 1936 and contested in 1937 from Bombay to the Central Legislative Assembly. He authored several books and essays. Some of them are: The Annihilation of Caste, Pakistan or the Partition of India, The Buddha and his Dhamma, The Evolution of Provincial Finance in British India, Administration and Finance of the East India Company, etc. His death anniversary is observed as Mahaparinirvan Din . His birth anniversary is celebrated as Ambedkar Jayanti or Bhim Jayanti on 14 April every year. Do you know? · A few months before he died, he converted to Buddhism · He considered the Right to Constitutional Remedy as the soul of the constitution. THINK! · Contributions of Dr. B.R. Ambedkar

QUESTION 31. Faizpur session of Indian National Congress is famous for which of the following reason? a) Formation of Swaraj Party b) Establishment of National Planning Committee c) Only session to be presided by Mahatma Gandhi d) First Session of Congress to be held in a village

Correct Answer: D Explanation Solution (d) Faizpur Session (December 1936): President - Jawaharlal Nehru It was the first session of Congress to be held in a village. Do you know? · Only session presided by Mahatma Gandhi was Belgaum in December 1924. THINK! · Important sessions of Congress

QUESTION 22. Radcliffe line represents the boundary between 1. India -Pakistan 2. India -Nepal 3. India -Myanmar 4. India -Bangladesh Select the correct code a) Only 1 b) 1 and 2 c) 1 and 3 d) 1 and 4

Correct Answer: D Explanation Solution (d) To fix the international boundaries between the two countries, the Boundary Commission was established chaired by Sir Cyril Radcliffe. The commission was to demarcate Bengal and Punjab into the two new countries. The boundary demarcation line is called Radcliffe line. It's western side serves as India-Pakistan border and eastern side serves as India- Bangladesh border. Do you know? · MacMohan line demarcates the border between India and china. However. China does not recognize it. THINK! · Important boundaries of the world

QUESTION 38. Hemis National park is believed to have the highest density of snow leopards of any protected area in the World. Which of the following statements are correct regarding Hemis National Park? 1. It is the only National Park in India which is north of the Himalayas. 2. It is the largest notified protected area in India. 3. Hemis National Park is India's only protected area inside the Palearctic ecozone. Select the code from below: a) 1 and 2 b) 2 and 3 c) 1 and 3 d) All of the above

Correct Answer: D Explanation Solution (d) Hemis National Park • Hemis National Park (or Hemis High Altitude National Park) is a high altitude national park in the eastern Ladakh region of the state of Jammu and Kashmir in India. • World famous for being the best place to see the snow leopard in the wild, it is believed to have the highest density of snow leopards of any protected area in the world. • It is the only national park in India north of the Himalayas, the largest notified protected area in India (and thus the largest national park of India), and is the second largest contiguous protected area after the Nanda Devi Biosphere Reserve and surrounding protected areas. • The park is home to a number of species of endangered mammals including the snow leopard (snow leopard is vulnerable as listed under IUCN) • Hemis National Park is India's only protected area inside the Palearctic ecozone, outside the Changthang Wildlife Sanctuary northeast of Hemis, and the proposed Tso Lhamo Cold Desert Conservation Area in North Sikkim. • The park is bounded on the north by the banks of the Indus River, and includes the catchments of Markha, Sumdah and Rumbak, and parts of the Zanskar Range. • The Tibetan wolf, the Eurasian brown bear (endangered in India), and the red fox are also present in Hemis. Think • Project Snow Leopard • Changthang Cold Desert Wildlife Sanctuary • Palearctic ecozone http://www.thehindu.com/sci-tech/energy-and-environment/how-homestays-are-saving-snow- leopards-in-ladakh/article21302533.ece

QUESTION 32. Which of the following is/are correct regarding elections to provincial assemblies in 1937? 1. The Congress Manifesto for elections was based on the 1935 Act. 2. Major agrarian reforms were undertaken under the Congress rule. 3. Congress Socialists and communists were opposed to office acceptance. Select the correct code a) 1 and 2 b) 1 and 3 c) Only 1 d) Only 3

Correct Answer: D Explanation Solution (d) In early 1937, elections to provincial assemblies were announced and once again the debate on the future strategy to be adopted began. There were sharp differences over these questions among the nationalists. The two sides of the debate soon got identified with the emerging ideological divide along the left and right lines. Nehru, Subhash, Congress socialists and communists were opposed to office acceptance and thereby in the working of the 1935 Act , because they argued that it would negate the rejection of the Act by the nationalists. As a counter-strategy, the leftists proposed entry into the councils with an aim to create deadlocks, thus making the working of the Act impossible (older Swarajist strategy). And, as a long-term strategy, they advocated an increased reliance on workers and peasants, integration of their class organisations into the Congress, thus imparting a socialist direction to the Congress and preparing for the resumption of a mass movement. The proponents of office acceptance argued that they were equally committed to combating the 1935 Act, but work in legislatures was to be only a short-term tactic since option of a mass movement was not available at the time, and mass struggle alone was capable of winning independence. The Congress manifesto reaffirmed total rejection of the 1935 Act , and promised release of prisoners, removal of disabilities on the basis of gender and caste, radical transformation of the agrarian system, substantial reduction of rent and revenue, scaling down of rural debts, cheap credit and right to form trade unions and to strike. Congress ministries were formed in Bombay, Madras, Central Provinces, Orissa, United Provinces, Bihar and later in NWFP and Assam also. The Congress ministries did much to ease curbs on civil liberties. There were certain basic constraints before the Congress ministries could undertake a complete overhaul of the agrarian structure by completely abolishing zamindari. In spite of the constraints, the Congress ministries managed to legislate a number of laws relating to land reforms, debt relief, forest grazing fee, arrears of rent, land tenures, etc. But most of these benefits went to statutory and occupancy tenants while sub-tenants did not gain much. Agricultural labourers did not benefit, as they had not been mobilised. Do you know? · Gandhi did not attend a single election meeting. · During this time, National Planning Committee was setup under Congress President Subhash Chandra Bose in 1938. This was India's first attempt to develop a national plan. THINK! · Reforms taken under 28 months of Congress rule · Struggle-Truce- Struggle (S-T-S) strategy

QUESTION 37. India is hosting the "Bodhi Parva: BIMSTEC Festival of Buddhist Heritage" as part of celebrations of 20th anniversary of BIMSTEC. Which of the following countries are parts of BIMSTEC? 1. India 2. Bangladesh 3. Bhutan 4. Myanmar 5. Sri Lanka Select the code from below: a) 1, 2 and 3 b) 3, 4 and 5 c) 1, 2, 3 and 4 d) All of the above

Correct Answer: D Explanation Solution (d) India is hosting the "Bodhi Parva: BIMSTEC Festival of Buddhist Heritage" as part of celebrations of 20th anniversary of BIMSTEC. The festival would have diverse components including an exhibition of international and Indian Buddhist art and architecture, discourses by eminent scholars and practitioners of Buddhism, guided meditation and chanting by Buddhist monks. Buddhist Circuits • The Buddhist Circuits are the Places of all High Significance Holy Sites of Buddhism; where Lord Buddha was born, attained Enlightenment, preached first Sermon and reached Nirvana. • Lumbini: Lumbini in southern Nepal is where Queen Mayadevi gave birth to Prince Siddhartha. It is just a short distance from the Shakya capital of Kapilavastu. • Bodhgaya: It was in Bodhgaya in Bihar, India that Prince Siddhartha found Enlightenment (nirvana) under the bodhi tree after meditating for 49 days. No longer a bodhisattva (mentor), he became Lord Buddha, the Enlightened One. • Sarnath: Buddha gave his first sermon at Sarnath after achieving enlightenment, about 10 km from the ancient holy city of Varanasi. The sermon, setting in motion the wheel of the teaching (dharamchakrapravartna) revealed to his followers the 4 noble truths, the concept of the 3 jewels of Buddha, Dharma and Sangha via the 8 fold path, for inner peace and enlightenment. It was here that the Buddha established his first disciples (sangha) to promote his new doctrine. The splendid Dhamekha Stupa at Sarnath was originally erected by King Ashoka, as was the famous lion capital pillar, now the proud symbol of India. • Kushinagar: At Kushinagar close to Gorakhpur in eastern Uttar Pradesh, India en route to Kapilavastu, Lord Buddha fell ill and left this world in 543 BC. His mortal remains were preserved in eight commemorative chortens, and then further distributed by King Ashoka into 84,000 stupas across his kingdom and beyond. The BIMSTEC member states are India, Bangladesh, Bhutan, Myanmar, Nepal, Sri Lanka and Thailand.

QUESTION 78. An anti-British Berlin-Indian Committee was formed in Berlin, Germany, in 1914. It became an organizational hub of Indian nationalism and revolutionary activity. Which of the following statements are correct regarding this committee? 1. The organisation was formed by Indian students and political activists who resided in the Germany. 2. The organisation was established with the aim to promote the cause of Indian Independence. 3. The Committee was formally disbanded in November 1918, with most of the members shifting their attention to the nascent Soviet Russia. Select the code from following: a) 1 and 2 b) 2 and 3 c) 1 and 3 d) All of the above

Correct Answer: D Explanation Solution (d) Indian - Berlin Committee During the World War I in 1914, an organisation was established in Germany, namely the Berlin Committee. After 1915, it was renamed the Indian Independence Committee. The organisation was formed by Indian students and political activists who resided in the Germany. The organisation was established with the aim to promote the cause of Indian Independence. In the beginning the orgnisation was called the Berlin-Indian Committee. Later, this Berlin-Indian Committee played an instrumental part in the Hindu-German Conspiracy. Virendranath Chattopadhyaya, Champakaraman Pillai and Abinash Bhattacharya were the key members of the committee. Think · How was Berlin Committee, India House and Reshmi Rumaal Tehreek connected?

QUESTION 47. Kalamkari is one of the most famous art forms of India. Which of the following statements are correct about Kalamkari? 1. It is a type of hand painted or block printed cotton textile. 2. Main hub of Kalamkari is Andhra Pradesh 3. It makes use of only natural colour of vegetable dyes. Select the code from following: a) 1 and 2 b) 2 and 3 c) 1 and 3 d) All of the above

Correct Answer: D Explanation Solution (d) Kalamkari • It is a type of hand-painted or block-printed cotton textile, produced in parts of India and Iran. • Kalamkari literally translates into "pen craft"; with 'kalam' meaning pen and 'kari' meaning art. It is among the most beautiful traditional Indian art forms and involves block printing or hand printing, typically done on pieces of cotton fabrics. • The unique feature of the Kalamkari art is that it makes use of only natural colours or vegetable dyes. • Kalamkari specifically depicts epics such as the Ramayana or Mahabharata. Origin • Kalamkari art and printing is concentrated primarily in Andhra Pradesh, particularly in Kalahasti and Machilipatnam, and a few other smaller regions of the state. • Srikalahasti was the main center of kalamkari art for a very important reason: it received a constant supply of clean river water. • Kalamkari art was the household occupation of several rural women and craftsmen in the ancient times and continues to be passed down from one generation to the next. • Andhra Pradesh is still the main hub of kalamkari printing in the country. Styles • Kalamkari art is available in two distinct styles: Machilipatnam and Srikalahasti. Both these centres are located in the state of Andhra Pradesh. Srikalahasti style of kalamkari • Srikalahasti is a small temple town found in the Chittoor district in Andhra. The main design inspiration for the Srikalahasti style is 'hindu mythology'. • The Srikalahasti style is characterized by one very important feature: free hand drawing. • The only other colours used for filling the outlined sketch (black colour) are those obtained from natural plants: indigo, green, red and vibrant yellow. Machilipatnam style of kalamkari • The craftsman creates his sketch and its key design features with the help of hand-carved blocks. • It was registered as one of the geographical indication from Andhra Pradesh under handicraft goods by Geographical Indications of Goods (Registration and Protection) Act, 1999. http://www.thehindu.com/news/national/andhra-pradesh/museum-that-captures-kalamkari-art/ article21615956.ece

QUESTION 50. Cabinet approved the establishment of International Training Centre for Operational Oceanography, as a Category-2 Centre (C2C) of UNESCO. In which of the places is it being established? a) Kolkata b) Chennai c) Cochin d) Hyderabad

Correct Answer: D Explanation Solution (d) Cabinet approved the establishment of International Training Centre for Operational Oceanography, as a Category-2 Centre (C2C) of UNESCO, in Hyderabad. Highlights • Purpose - To establish a training centre towards development of capacity for the countries on the Indian Ocean Rim (IOR), African countries bordering the Indian and Atlantic Oceans, small island countries under the framework of UNESCO • Operational oceanography - It is an activity of conducting systematic oceanographic studies towards providing information services to various sectors viz. fisherman, disaster management, shipping, ports, coastal states, navy, coast guard, environment, offshore industries for conducting their day-to- day operations. • It is intended to improve skill of students and other participants which will increase the employment opportunities within and outside India. • It is also expected to increase ancillary development leading to employment generation in India Think • Indian National Centre for Ocean Information Services (INCOIS) • Sustainable Development Goal-14 http://pib.nic.in/newsite/PrintRelease.aspx?relid=174372

QUESTION 69. Consider the following statements regarding 'Lucknow Pact': 1. Lucknow pact was signed between Congress and Muslim League 2. The Pact was signed in 1916 between Tilak and Jinnah representing Congress and Muslim League respectively. 3. Extremist and Moderates reunited in 1916 at Lucknow session. Which of the above statements are correct? a) 1 and 2 b) 2 and 3 c) 1 and 3 d) All of the above

Correct Answer: D Explanation Solution (d) Lucknow Pact , (December 1916), agreement made by the Indian National Congress headed by Maratha leader Bal Gangadhar Tilak and the All-India Muslim League led by Muhammad Ali Jinnah; it was adopted by the Congress at its Lucknow session on December 29 and by the league on Dec. 31, 1916. The meeting at Lucknow marked the reunion of the moderate and radical wings of the Congress. The pact dealt both with the structure of the government of India and with the relation of the Hindu and Muslim communities. On the former count, the proposals were an advance on Gopal Krishna Gokhale's "political testament." Four-fifths of the provincial and central legislatures were to be elected on a broad franchise, and half the executive council members, including those of the central executive council, were to be Indians elected by the councils themselves. Except for the provision for the central executive, these proposals were largely embodied in the Government of India Act of 1919. The Congress also agreed to separate electorates for Muslims in provincial council elections and for weightage in their favour (beyond the proportions indicated by population) in all provinces except the Punjab and Bengal, where they gave some ground to the Hindu and Sikh minorities. This pact paved the way for Hindu-Muslim cooperation in the Khilafat movement and Mohandas Gandhi's noncooperation movement from 1920. Think · When did the conflict between Muslim league and Congress started? · Khilafat Movement

QUESTION 92. A photon is a particle of light defined as a discrete bundle (or quantum) of electromagnetic (or light) energy. Consider the following statements regarding 'Photon': 1. Photon behaves like a particle and a wave, simultaneously. 2. They have zero mass and rest energy 3. They are electrically neutral Which of the given statement/s is/are correct? a) 1 only b) 2 and 3 c) 1 and 3 d) 1, 2 and 3

Correct Answer: D Explanation Solution (d) On December 18, 1926, in a letter to the journal Nature, Gilbert Newton Lewis coined the term "photon". A photon is a particle of light defined as a discrete bundle (or quantum) of electromagnetic (or light) energy. Photons are always in motion and, in a vacuum (a completely empty space), have a constant speed of light to all observers. Photons travel at the vacuum speed of light (more commonly just called the speed of light) of c= 2.998 x 108 m/s. According to the photon theory of light, photons: · behave like a particle and a wave, simultaneously · move at a constant velocity, c = 2.9979 x 108 m/s (i.e. "the speed of light"), in empty space · have zero mass and rest energy · Carry energy and momentum, which are also related to the frequency (nu) and wavelength (lamdba) of the electromagnetic wave, as expressed by the equation E = h nu and p = h / lambda. · Can be destroyed/created when radiation is absorbed/emitted. · Can have particle-like interactions (i.e. collisions) with electrons and other particles · Are electrically neutral http://www.thehindu.com/children/lets-call-it-photon/article21830971.ece

QUESTION 72. Which of the following causes are correct regarding the growth of extremism in early 1900s in congress? 1. Dissatisfaction with passive activities of Moderates 2. Growth of western education 3. Reaction to increasing westernization of Indians 4. Reactionary policies of Curzon Select the code from following: a) 1, 2 and 3 b) 1 and 3 c) 2, 3 and 4 d) All of the above

Correct Answer: D Explanation Solution (d) Prominent Reasons for the rise of Extremism: · Some of the Congress leaders, specially the younger element, were greatly disappointed with the attitude of the British Government towards their demand. The Indian Council Act of 1892 which was enacted as a sort of concession to the people of India did not concede anything substantial. · As a result of the socio-religious reform movements of the late nineteenth century, a large number of young Congress leaders like Bal Gangadhar Tilak, Lai Lajpat Rai, B.C. Pal were convinced of the glorious culture of India and wanted to revive the same instead of aping the western civilization. They emphasised that most of country's ills were due to foreign rule and promoted a feeling of patriotism among the Indian people. · The misery and suffering caused by the famines of 1896-98 and 1899-1901, and bubonic plague around the same time also greatly agitated people. They were greatly dissatisfied with the measures taken by the Government to deal with the situation. · The partition of Bengal in 1905 also contributed to the growth of extremism. · The imperialist policies followed by Lord Curzon during his viceroyalty from 1899-1905 gave a further fillip to the extremist movement in India. · As a result of Western education, the Congress leaders learnt that even countries like America, Germany, Italy, France, and Britain had to resort to force to gain independence or preserve democracy. · Certain events in foreign lands during the closing years of the nineteenth century and the beginning of the twentieth century also provided inspiration to the Indian, and imparted great vigour to the national movement. In 1896 Abyssinia, and Africa Kingdom, succeeded in defeating a powerful country like Italy. In 1904-05, Japan, a tiny Island country inflicted a defeat on Russia, one of the largest state. Think · Prominent extremist leaders

QUESTION 64. Who of the following established the first Provisional Government of India at Kabul in Afghanistan as a government-in-exile of Free Hindustan, with himself as President? a) Maulvi Barkatullah b) Lala Hardayal c) Shyamji Krishnavarma d) Raja Mahendra Pratap

Correct Answer: D Explanation Solution (d) Raja Mahendra Pratap Singh (1 December 1886 - 29 April 1979) was a freedom fighter, journalist, writer, and Marxist revolutionary social reformist of India and President of first Provisional Government of India. He also formed the Executive Board of India in Japan in 1940 during the Second World War. He also took part in the Balkan War in the year 1911 along with his fellow students of MAO college. In recognition of his services, the government of India issued postage stamps in his honor. On 1 December 1915 during World War I (his 28th birthday) Pratap established the first Provisional Government of India at Kabul in Afghanistan as a government-in-exile of Free Hindustan, with himself as President, Maulavi Barkatullah as Prime Minister, and Maulavi Abaidullah Sindhi as Home Minister, declaring jihad on the British. Anti-British forces supported his movement, but because of obvious loyalty to the British, the Amir kept on delaying the expedition to overthrow British rule in India. Due to his revolutionary ideas Pratap had a good relationship with Lenin, who invited him to Russia after its liberation and welcomed him. By this time he had become a real threat to British rule in India, and the British Government of India put a bounty on his head, attacked his entire estate, and declared him a fugitive, causing him to flee to Japan in 1925. Think · First World War · India House in London · Indian contribution in first World War

QUESTION 82. Who of the following personalities renounced his knighthood due to their response towards Home Rule League Movement? a) Rabindranath Tagore b) Sir Sayyad Khan c) Mahatma Gandhi d) Sir S subramaniya Aiyyar

Correct Answer: D Explanation Solution (d) Sir Subbier Subramania Iyer was an Indian lawyer, jurist and freedom fighter who, along with Annie Besant, founded the Home Rule Movement. He was popularly known as the "Grand Old Man of South India". Subramania Iyer was born in the Madurai district of Madras Presidency. On completion of his schooling in Madura, Subramania Iyer qualified as a lawyer from the University of Madras, and went on to practice as a lawyer in Madurai and Madras, before being appointed a Judge of the Madras High Court, in 1891. He also served as the first Indian Chief Justice of the Madras High Court, before retiring in 1907. Do you know? · Rabindranath Tagore gave up his knighthood to protest against 'Jallianwala Bagh Massacre'.

QUESTION 14. Which of the following statements is/are correct w.r.t Bardoli Satyagraha? 1. The main reason for protest was use of force and oppression by Zamindars. 2. Mahatma Gandhi was called to lead the movement. Select the correct code a) Only 1 b) Only 2 c) 1 and 2 d) None

Correct Answer: D Explanation Solution (d) The Bardoli taluqa in Surat district had witnessed intense politicization after the coming of Gandhi on the national political scene. The movement sparked off in January 1926 when the authorities decided to increase the land revenue by 30 per cent . The Congress leaders were quick to protest and a Bardoli Inquiry Committee was set up to go into the issue. The committee found the revenue hike to be unjustified. In February 1926, Vallabhbhai Patel was called to lead the movement . Under Patel, the Bardoli peasants resolved to refuse payments of the revised assessment until the Government appointed an independent tribunal or accepted the current amount as full payment. To organize the movement, Patel set up 13 chluzvanis or workers' camps in the taluqa. Bardoli Satyagraha Patrika was brought out to mobilize public opinion. An intelligence wing was set up to make sure all the tenants followed the movement's resolutions. Those who opposed the movement faced a social boycott. Special emphasis was placed on the mobilization of women. K.M. Munshi and Lalji Naranji resigned from the Bombay Legislative Council in support of the movement. By August 1928, massive tension had built up in the 'area. There were prospects of a railway strike in Bombay. Gandhi reached Bardoli to stand by in case of any emergency. The Government was looking for a graceful withdrawal now. It set the condition that first the enhanced rent be, paid by all the occupants (not actually done). Then, a committee went into the whole affair and found the revenue hike to be unjustified and recommended a rise of 6.03 per cent only. Do you know? · The women of Bardoli gave Vallabhai Patel the title of "Sardar". · The movement is criticized as it was focused on rich and middle class farmers and neglected poor farmers and the problem of Hali Pratha (bonded labour system) THINK! · Important peasant movements


Conjuntos de estudio relacionados

Cell and Molecular Bio Chapter 6 Practice Problems

View Set

Chapter 7- Anatomy, Physiology and Medical Terminology

View Set

SmartyPANCE - Make me Ortho smart

View Set

Major Mountains, Ranges and other features of the World, School of Dad

View Set

Module 6 Other Insurance Policies

View Set